Krok-2 Pediatrics Base with Highlights & Explanation

The bellow material is aimed to improve the study and understanding of the Krok 2 2019 Pediatrics Bases with the use of highlights for relevant key works, links to relevant study materials, reference images and explanations to individual questions aimed at making remembrance of questions easier and also understanding of new questions for the upcoming Step 2 exam.

 

 

1. Routine examination of an otherwise healthy child with a history of bronchial asthma reveals AP of 140/90 mm Hg. The most likely cause of the hypertension is:

A. *Renal disease
aaaaaaaaaaaaaaaaaaa
B. Theophylline toxicity → cause HYPOTENSION , Tachycardia, ↓K+, ↑Ca 2

C. Chronic lung disease

D. Coarctation of the aorta

E. Obesity

 

Explanation:

Renal disease → here as the pt. had bronchial asthma (hypersensitivity reaction type I) so the pt could have due that glomerulonephritis (which is also hypersensitivity reaction Type III) , due to this may induce hypertension.

Coarctation of the aorta → HTN differs between upper and Lower extremities.

 

2. The child is 11 m.o. He suffers from nervous-arthritic diathesis. The increased synthesis of what acid is pathogenic at nervous-arthritic diathesis?

A. *Uric acid

B. Acetic acid

C. Phosphoric acid

D. Hydrochloric acid

E. Sulfuric acid

 

Examination:

Uric acid is an altered reactivity of the nervous system caused by the inheritance of the morphofunctional features of the organism that determine the breakdown of the breakdown and synthesis of purine nucleotides these people are sick with gout, obesity, nephritis, urolithiasis, diabetes, cholelithiasis, early atherosclerosis. Uric acid is responsible for this.

Neuro-arthritic diathesis is characterized by metabolic disorders – purine, lipid, and carbohydrate. … Urine acid diathesis is characterized by the accumulation of uric acid diathesis and its salts in biological fluids. Oxalic acid disorder is very often with this type of diathesis.

* Diathesis a hereditary or constitutional predisposition to a disease or other disorder

 

3. A 10-year-old child complains of fever (temperature is 39oC), frequent painful urination (pollakiuria). Urine test: proteinuria (0,066 g/L), leukocytouria (entirely within eyeshot), bacteriuria (105 colony forming units/mL). What is the most probable diagnosis?

A. *Acute pyelonephritis

B. Acute glomerulonephritis

C. Dysmetabolic nephropathy

D. Acute cystitis

E. Urolithiasis

 

Explanation:

Acute pyelonephritis there is symptoms of infection , so Bacteria & WBC is seen in urine , in addition to that very mild proteinuria comparing with GN very high & frequent painful urination.

Acute glomerulonephritis mostly post streptococcal after URTI / tonsillitis and will have hematuria

Dysmetabolic nephropathy renal disease due metabolic disorder.

Acute cystitis should tell site of pain in pubis area or lower abdomen and Dysuria (Burning sensation on urination). As well as Pasternatsky sign will be – ve , which is more specific for pyelonephritis .

Urolithiasis urinary tract stones , usu pain and hematuria .

 

4. A patient with acute respiratory viral infection (3rd day of disease) complains of pain in lumbar region, nausea, dysuria, oliguria. Urinalysis – hematuria (100-200 RBC in eyeshot spot), specific gravity – 1002. The blood creatinin level is 0,18 millimole/l, potassium level – 6,4 millimole/l. Make the diagnosis:

A. *Acute interstitial nephritis

B. Acute renal failure

C. Acute glomerylonephritis

D. Acute cystitis

E. Acute renal colic

 

Explanation:

↓ specific gravity, ↑ blood creatinin and potassium( potassium is reabsorbed ad the tubules)

Dysuria indicates affection bellow the kidney level/ glomerulus

Blood creatinin = 53 – 106 micromole/liter OR 0.053 – 0.106 millimole/liter

Acute interstitial nephritis (AIN) defines a pattern of renal injury usually associated with an abrupt deterioration in renal function characterized histopathologically by inflammation and edema of the renal interstitium.

Symptoms of interstitial nephritis
  • Increased urine output.
  • Blood in your urine or dark urine.
  • Nausea or vomiting.
  • Fever or rash.
  • Elevated blood pressure.
  • Changes in mental status, such as drowsiness or confusion.
  • Swelling of any area of your body.
  • Sudden weight gain (this can be caused by extra fluid in the body).

 

5. A 4-month-old girl with blond hair and blue eyes has “mousy” odor of sweat and urine, delayed psychomotoric development. The most typical laboratory data for this disorder is:

A. *Positive urine ferric chloride test

B. High level of oxyproline in urine

C. High level of glycosaminoglycanes in urine

D. High concentration of chlorides in sweat

E. Low level of thyroid gland hormones in blood

 

Explanation:

mousy” odor of sweat and urine indicates Phenylketonuria

Phenylketonuria (PKU) is an inborn error of metabolism that results in decreased metabolism of the amino acid phenylalanine. Untreated, PKU can lead to intellectual disability, seizures, behavioral problems, and mental disorders. It may also result in a musty smell and lighter skin.

Other names: Phenylalanine hydroxylase deficiency, PAH deficiency, Følling disease

PKU signs and symptoms can be mild or severe and may include:
  • A musty odor in the breath, skin or urine, caused by too much phenylalanine in the body.
  • Neurological problems that may include seizures.
  • Skin rashes (eczema)

 

6. A girl is 12 y.o. Yesterday she was overcooled. Now she is complaining on pain in suprapubic area, frequent painful urination by small portions, temperature is 37,8o. Pasternatsky symptom is negative. Urine analysis: protein – 0,033 g/L, WBC- 20-25 in f/vis, RBC- 1-2 in f/vis. What diagnosis is the most probable?

A. *Acute cystitis

B. Dysmetabolic nephropathy

C. Acute glomerulonephritis

D. Acute pyelonephritis

E. Urolithiasis

 

7. A 12 y.o. child with acute glomerulonephritis presented with hypertensive syndrome during first days of the disease. What is the role of angiotesin II in the pathogenesis?

A. *Intensifies production and secretion of aldosterone

B. Increases heart output

C. Infibits deppresive action of prostaglandins

D. Increases erythropoetin production

E. Increases renine level

 

Exp:

The overall effect of angiotensin II is to increase blood pressure, body water and sodium content. Angiotensin II has effects on: Blood vessels – it increases blood pressure by causing constriction (narrowing) of the blood vessels. … The kidneys: it increases sodium retention and alters the way the kidneys filter blood.

Image result for role of angiotensin II

 

8. A 7 y.o. boy has been treated in a hospital for a month. At the time of admission he had evident edemata, proteinuria – 7,1 g/L, protein content in the daily urine – 4,2 g. Biochemical blood analysis reveals permanent hypoproteinemia (43,2 g/L), hypercholesterolemia (9,2 mmol/L). What variant of glomerulonephritis is the most probable?

A. *Nephrotic

B. Nephritic

C. Isolated urinary

D. Hematuric

E. Mixed

 

Image result for nephrotic vs nephritic

Image result for nephrotic vs nephritic

 

9. 2 weeks after recovering from angina an 8-year-old boy developed edemata of face and lower limbs. Objectively: the patient is in grave condition, AP- 120/80 mm Hg. Urine is of dark brown colour. Oliguria is present. On urine analysis: relative density – 1,015, protein – 1,2 g/l, RBCs are leached and cover the whole vision field, granular casts – 1-2 in the vision field, salts are represented by urates (big number). What is the most likely diagnosis?

A. *Acute glomerulonephritis with nephritic syndrome

B. Acute glomerulonephritis with nephrotic syndrome

C. Acute glomerulonephritis with nephrotic syndrome, hematuria and hypertension

D. Acute glomerulonephritis with isolated urinary syndrome

E. Nephrolithiasis

 

Explanation:

Image result for nephrotic vs nephritic

 

10. An infant is 2 d.o. It was full-term born with signs of intrauterine infection, that’s why it was prescribed antibiotics. Specify, why the gap between antibiotic introductions to the new-born children is longer and dosage is smaller compared to the older children and adults?

A. *The newborns have a lower level of glomerular filtration → most antibiotic clearance through kidney and Neonate still have Low GFR , so need to lower dose of drug and increase intervals .

B. The newborns have lower concentration of protein and albumins in blood

C. The newborns have reduced activity of glucuronil transferase

D. The newborns have diminished blood pH

E. The newborns have bigger hematocrit

 

11. A 10 y.o. child who is at oligoanuretic stage of acute renal insufficiency has got sensations of pricking in the mucous membrane of oral cavity and tongue, extremities numbness, reduced reflexes, respiratory disturbance, arrhythmia. What are these symptoms caused by?

A. *Hyperkaliemia → ↑ K+

B. Hyponatremia → ↓ Na+

C. Hyperazotemia → ↑ nitrogen waste in blood

D. Acidosis → if metabolic PH ↓, HCO3 ↓, PCO2

E. Alkalosis → if metabolic PH ↑ HCO3 ↑, PCO2↑

 

Exp:

oligoanuria (uncountable) (pathiology) A condition, lying between anuria and oligouria, in which less than 100ml of urine is produced per day quotations

Image result for hyperkalemia symptomsImage result for hyperkalemia symptoms

 

12. An 11-year old girl was taken by an acute disease: she got pain in the lumbar region, nausea, vomiting, frequent urination, body temperature 39oC. Objectively: the abdomen is soft, painful on palpation in the lumbar region. Common urine analysis revealed considerable leukocyturia, bacteriuria. The urine contained colibacilli. What is the most likely diagnosis?

A. *Acute pyelonephritis

B. Acute appendicitis

C. Chronic glomerulonephritis

D. Acute vulvovaginitis

E. Acute glomerulonephritis

 

13. A 4-year-old boy in 2 weeks after the tonsillitis had edema, headache, vomiting three times per day. On physical exam: rise of blood pressure, urine is of meat slops color. What is the most probable diagnosis?

A. *Glomerulonephritis

B. Pyelonephritis

C. Interstitial nephritis

D. Cystitis

E. Urethritis

 

14. A patient complains about sudden onsets of paroxysmal pain in the right lumbar region. 2 hours after the onset the patient had hematuria. Plain radiograph of the lumbar region shows no pathological shadows. USI reveals pyelocaliectasis on the right, the left kidney is normal. What is the most likely diagnosis?

A. *Renal colic

B. Acute appendicitis

C. Bowel volvulus

D. Torsion of the right ovary cyst

E. Right renal pelvis tumour

 

Exp:

pyelocaliectasis. [″ + kalyx, cup, + ektasis, dilation] Dilation of the pelvis and calices of the kidney.

 

15. A 12-year-old boy has a 6-year history of insulin-dependent diabetes. The disease is labile. Since recently there have been periodical rises in blood pressure. Microalbuminuria test gave positive results. The patient’s condition corresponds with the following stage of diabetic nephropathy:

A. *Stage III – early-stage nephropathy

B. Stage I – renal hypertrophy and hyperfunction

C. Stage II – histological changes in the kidneys

D. Stage IV – advanced clinical nephropathy

E. V stage – chronic renal failure

 

Exp:

Microalbuminuria is indicative for stage 3 or 4 but due to the childs age chronological history indicates stage 3. See table bellow

Stages

A doctor may break down the stages of kidney disease, depending on the GFR, which also represents the percentage of effective kidney function.

Stage 1: Kidney damage present but normal kidney function and a GFR of 90% or above.

Stage 2: Kidney damage with some loss of function and a GFR of 60–89%.

Stage 3: Mild to severe loss of function and a GFR of 30–59%.

Stage 4: Severe loss of function and GFR of 15–29%.

Stage 5: Kidney failure and a GFR of under 15%.

Image result for stage of diabetic nephropathy

liable to change; easily altered.
“persons whose blood pressure is more labile will carry an enhanced risk of heart attack”
  • of or characterized by emotions which are easily aroused, freely expressed, and tend to alter quickly and spontaneously.

 

16. Examination of a 13-year-old girl revealed acute glomerulonephritis, nephritic syndrome at the initial stage without renal dysfunction. What is the main drug of choice for the basic therapy of this patient?

A. *Antibiotic

B. Prednisolone

C. Curantyl

D. Heparin

E. Saluretic

 

17. An 8-month-old baby has decreased appetite, pale skin, enlarged right side of abdomen. Palpation the right side of abdomen reveals a dense elastic tumour like formation 10×7 cm large. There is a positive ballotement sign. What is the most likely diagnosis?

A. *Nephroblastoma (Wilms’ tumour)

B. Liver tumour

C. Congenital hydronephrosis

D. Colon pathology

E. Intestinal tumour

 

Exp:

Wilmstumor is a rare kidney cancer that primarily affects children. Also known as nephroblastoma, it’s the most common cancer of the kidneys in children. Wilmstumor most often affects children ages 3 to 4 and becomes much less common after age 5.

Ballotement is a sharp upward pushing against the uterine wall with a finger inserted into the vagina for diagnosing pregnancy by feeling the return impact of the displaced fetus also : a similar procedure for detecting a floating kidney.

 

18. From urine of a 14-year-old boy with the exacerbation of secondary obstructive pyelonephritis Pseudomonas aeruginosa was isolated with a titer of 1000000 microbes per 1 ml. Which antibiotic is most advisable to be administered in this case?

A. *Ciprofloxacin

B. Ampicillin

C. Cefazolinum

D. Azithromycin

E. Chloramphenicol

 

Exp:

Antibiotics should include an aminoglycoside and an antistaphylococcal agent. If Pseudomonas species are considered or demonstrated, an antipseudomonal beta-lactam antibiotic should be added. For enterococci, an aminoglycoside and ampicillin are recommended.

Ciprofloxacin

Ciprofloxacin is an antibiotic used to treat a number of bacterial infections. This includes bone and joint infections, intra abdominal infections, certain type of infectious diarrhea, respiratory tract infections, skin infections, typhoid fever, and urinary tract infections, among others.

 

19. A young man has painful indurations in the peripapillary regions of both mammary glands. The most reasonable action will be:

A. *To leave these indurations untouched

B. To remove them

C. To cut and drain them

D. To take an aspirate for bacterial inoculation and cytology

E. To administer steroids locally

 

Explanation:

Induration : The hardening of a normally soft tissue or organ, especially the skin, due to inflammation, infiltration of a neoplasm,or accumulation of blood.
maturity changes or one symptoms of breast malignization ??

 

20. A 7 d.o. boy is admitted to the hospital for evaluation of vomiting and dehydration. Physical examination is otherwise normal except for minimal hyperpigmentation of the nipples. Serum sodium and potassium concentrations are 120 meq/L and 9 meq/L respectively. The most likely diagnosis is:

A. *Congenital adrenal hyperplasia

B. Pyloric stenosis

C. Secondary hypothyroidism

D. Panhypopituitarism

E. Hyperaldosteronism

 

Exp:

Serum potassium is narrow (3.5 to 5.5 mEq/L)

A normal blood sodium level is between 135 and 145 milliequivalents per liter (mEq/L)

 

21. A 12-year-old girl applied to doctor with complaints of swelling on the front part of the neck.The doctor diagnosed hyperplasia of the thyroid gland of the second degree, euthyroidism. Ultrasound suspected autoimmune thyroiditis. Blood was taken for titre of antibodies to thyroglobulin. What titre of antibodies will be diagnostically important?

A. *1:100

B. 1:50

C. 1:150

D. 1:200

E. 1:250

 

22. A 14-year-old girl has been presenting with irritability and tearfulness for about a year. A year ago she was also found to have diffuse enlargement of the thyroid gland (II grade). This condition was regarded as a pubertal manifestation, the girl didn’t undergo any treatment. The girl’s irritability gradually gave place to a complete apathy. The girl got puffy face, soft tissues pastosity, bradycardia, constipations. Skin pallor and gland density progressed, the skin got a waxen hue. What disease may be assumed?

A. *Autoimmune thyroiditis

B. Diffuse toxic goiter

C. Thyroid carcinoma

D. Subacute thyroiditis

E. Juvenile basophilism

 

23. A 2 m.o. child with birth weight 5100 g has jaundice, hoarse cry, umbilical hernia, physical development lag. Liver is +2 cm enlarged, spleen is not enlarged. In anamnesis: delayed falling-away of umbilical cord rest. In blood: Hb- 120 g/L, erythrocytes – 4,5*1012/L, ESR- 3 mm/h. Whole serum bilirubin is 28 mcmole/L, indirect – 20 mcmole/L, direct – 8 mcmole/L. What is the most probable diagnosis?

A. *Congenital hypothyreosis → congenital Hypothyroidism

B. Congenital hepatitis

C. Hemolitic anemia

D. Conjugated jaundice

E. Cytomegalovirus infection

 

Image result for congenital hypothyroidism

 

24. A 40 h.o. child age has hyperosthesia, CNS depression, dyspepsia. Sepsis is suspected. What should the differential diagnosis be made with?

A. *Hypoglycemia

B. Hypocalcemia

C. Hyperbilirubinemia

D. Hyperkaliemia

E. Hypomagnesemia

 

25. A 6 y.o child complains of thirst, polyuria, increased appetite for 2 months with weight loss for 3kg. There has been nocturnal enuresis during last week. On examination: hyperglycemia 14 mol/L. The diagnosis is diabetis mellitus I type. What is the genesis of this disease?

A. *Autoimmune

B. Viral

C. Bacterial

D. Neurogenic

E. Virus-bacterial

26. Examination of a 12 year old child revealed diffuse thyroid enlargement of the II degree. Heart auscultation revealed dullness of heart sounds, heart rate was 64/min. The child has frequent constipations, anemia. Concentration of thyreoglobulin antibodies is increased. What disease might have caused such symptoms?

A. *Autoimmune thyroiditis

B. Diffuse toxic goiter

C. Thyroid carcinoma

D. Thyroid hyperplasia

E. Endemic goiter

27. A 9 year old boy had acute respiratory viral infection. After it there appeared polydipsia, polyuria, weakness, nausea. Examination revealed the following symptoms: mental confusion, dry skin, soft eyeballs, Kussmaul’s respiration, acetone smell from the mouth, muffled heart sounds, soft and painless abdomen. Blood sugar was 19 millimole/l. What acute condition is it?

A. *Ketoacidotic coma

B. Hyperosmolar coma

C. Cerebral coma

D. Hepatic coma

E. Acute renal insufficiency

28. A 16-year-old girl had had polyuria, polydipsia for 2 months. She had lost 8 kg with a good appetite. The patient was urgently hospitalized for abdominal pain and nausea. Examination revealed glycemia at the rate of 18 mmol/l, glycosuria at the rate of 24 g/l. Insulin and infusion of isotonic solutions of sodium chloride and glucose eliminated these problems, including thirst. What is the most likely diagnosis?

A. *Diabetes mellitus type 1

B. Diabetes mellitus type 2

C. Renal glycosuria

D. Diabetes insipidus

E. Secondary (symptomatic) diabetes

29. An 8-year-old child with a 3-year history of diabetes was hospitalized in hyperglycemic coma. Specify the initial dose of insulin to be administered:

A. *0,1-0,2 U/kg of body weight per hour

B. 0,05 U/kg of body weight per hour

C. 0,2-0,3 U/kg of body weight per hour

D. 0,3-0,4U/kg of body weight per hour

E. 0,4-0,5 U/kg of body weight per hour

30. A 10-year-old boy had a case of hepatitis B four years ago. The formation of hepatic cirrhosis in the patient was suspected. What additional investigation can clarify the diagnosis?

A. *Liver needle biopsy

B. Proteinogram

C. Echocholecystography

D. Markers of hepatitis type B

E. Transaminase level measurement

31. A 6-year-old child complains of frequent liquid stool and vomiting. On the 2nd day of disease the child presented with inertness, temperature rise up to 38.2oC, Ps- 150/min., scaphoid abdomen, palpatory painful sigmoid colon, defecation 10 times a day with liquid, scarce stool with mucus and streaks of green. What is the provisional diagnosis?

A. *Shigellosis

B. Salmonellosis

C. Escherichiosis

D. Intestinal amebiasis

E. Yersiniosis

32. A patient has been hospitalised. The onset of the disease was gradual: nausea, vomiting, dark urine, аcholic stool, yellowness of the skin and scleras. The liver is enlarged by 3 cm. Jaundice increased on the 14th day of the disease. The liver diminished in size. What complication of viral hepatitis caused deterioration of the patient’s condition?

A. *Hepatic encephalopathy

B. Meningitis

C. Relapse of viral hepatitis

D. Cholangitis

E. Infectious-toxic shock

33. A 15-year-old patient was delivered to an infectious diseases unit on the 3rd day of illness with complaints of headache, pain in lumbar spine and gastrocnemius muscles, high fever, chill. Objectively: condition of moderate severity. Scleras are icteric. Pharynx is hyperemic. Tongue is dry with dry brown coating. Abdomen is distended. Liver is enlarged by 2 cm. Spleen is not enlarged. Palpation of muscles, especially gastrocnemius muscles, is painful. Urine is dark in color. Stool is normal in color. The most likely diagnosis is:

A. *Leptospirosis

B. Hepatitis A

C. Malaria

D. Infectious mononucleosis

E. Yersiniosis

34. A 13-year-old male patient developed a condition that had a stormy clinical course: chills, fever up to 39oC, vomiting, epigastric pain, diarrhea with watery smelly feces. 6 hours before, he ate a raw egg, fried potatoes with stewed meat, and drank some juice. What pathogen is likely to have caused this condition?

A. *Salmonella

B. Colibacillus

C. Campylobacter

D. Shigella

E. Vibrio cholera

35. A 12-year-old boy presents with nausea, frequent repeated vomiting that first occurred after eating canned vegetables. Objectively: the patient has dry mucous membranes, muscular hypotonia, anisocoria, mydriasis, dysphagia and dysarthria. What is the most likely diagnosis?

A. *Botulism

B. Shigellosis

C. Salmonellosis

D. Cholera

E. Yersiniosis

36. A 16-year-old male patient consulted a doctor abut sore throat, fever up to 38.2oC. A week before, the patient had quinsy, didn’t follow medical recommendations. On examination, the patient had forced position of his head, trismus of chewing muscles. Left peritonsillar region is markedly hyperemic, swollen. What is the provisional diagnosis?

A. *Left-sided peritonsillar abscess

B. Meningitis

C. Phlegmonous tonsillitis

D. Pharyngeal diphtheria

E. Tonsil tumor

37. A 7-year-old boy had complained of headache, nausea, and fatigue for 3 weeks. His condition gradually deteriorated, headache and general weakness progressed. The boy had bronchitis at the age of 3. His father has a history of pulmonary tuberculosis. Objectively: body temperature 37.5oC, conscious, lies supine, with the hip and knee flexed to 90 degrees, nuchal rigidity +6 cm, partial ptosis of the right eyelid, the dilated right pupil. General hyperalgesia is present. Liquor: transparent, pressure – 400 mm of water column, protein – 1.5%, cytosis – 610/3 with predominant lymphocytes, sugar – 1.22 mmol/l, chlorides – 500 mmol/l. What is the most likely diagnosis?

A. *Tuberculous meningitis

B. Secondary purulent meningitis

C. Epidemic cerebrospinal meningitis

D. Serous meningitis

E. Pneumococcal meningitis

38. Medical examination of a 17-year-old student revealed generalized lymphadenopathy mainly affecting the posterior cervical, axillary and ulnar lymph nodes. There are multiple injection marks on the elbow bend skin. The man denies taking drugs, the presence of injection marks ascribes to influenza treatment. Blood count: RBCs – 3.2×1012/l, Hb – 100 g/l, WBCs – 3.1×109/l, moderate lymphopenia. What study is required in the first place?

A. *ELISA for HIV

B. Immunogram

C. Sternal puncture

D. X-ray of lungs

E. Lymph node biopsy

39. A patient complains of frequent, bulky, frothy stools with greenish mucus, cramping pain in the umbilical region, abdominal murmur, body temperature at the rate of 39oC. The patient associates the disease with consumption of soft-boiled eggs. What is the most likely pathogen?

A. *Salmonella

B. Yersinia

C. Shigella

D. Enteropathogenic E.Coli

E. Vibrio cholerae El Tor

40. A patient was delivered to the admission ward of the infectious disease department. The disease had started acutely on the background of normal temperature with the appearance of frequent, liquid, profuse stool without pathological impurities. Diarrhea was not accompanied by abdominal pain. 12 hours later there appeared recurrent profuse vomiting. The patient rapidly developed dehydration. What is the most likely diagnosis?

A. *Cholera

B. Shigellosis

C. Staphylococcal food toxicoinfection

D. Salmonellosis

E. Campylobacteriosis

41. On the second day after preventive vaccination a 2-year-old boy presented with abdominal pain without clear localization, body temperature rose up to 38 oC. On the third day the child got red papular haemorrhagic eruption on the extensor surfaces of limbs and around the joints. Knee joints were edematic and slightly painful. Examination of other organs and systems revealed no pathological changes. What is the most likely diagnosis?

A.

42. A 10-year-old patient complains of paroxysmal abdominal pain, frequent liquid stools up to 10 times a day. Throughout the first 3 days he had a fever, since the 2nd day of disease there were scant liquid stools mixed with mucus. On palpation: tenderness of all colon segments. Sigmoid colon was found spastic. What is your provisional diagnosis?

A. *Acute shigellosis

B. Intestinal amebiasis

C. Salmonellosis

D. Cholera

E. Balantidiasis

43. Half an hour after a 14-year-old female had had some custard cake, she experienced lancinating abdominal pain, nausea, vomiting. Objectively: body temperature – 36.0 oC, pale skin, breathing rate – 20/min, Ps – 100/min. AP – 95/65 mm Hg, loud cardiac sounds. Dry tongue. Abdomen was painful in its epigastrial part, there were no signs of peritoneum irritation. What is the first measure to be taken?

A. *Gastric lavage

B. Administration of an enterosorbent

C. Injection of Cerucal

D. Intravenous rehydratation

E. Antibiotic therapy

44. An 8-year-old boy fell ill acutely: he presents with fever, weakness, headache, abdominal pain, recurrent vomiting, then diarrhea and tenesmus. Stools occur 12 times daily, are scanty, contain a lot of mucus, pus, streaks of blood. His sigmoid gut is tender and hardened. What is your diagnosis?

A. *Shigellosis

B. Salmonellosis

C. Cholera

D. Staphylococcal gastroenteritis

E. Escherichiosis

45. A nurse of the kindergarten was taken to the hospital with complaints of acute pain in peryumbilical region, convulsions of lower limbs, multiple bile vomiting, frequent watery foul feces of green color in huge amounts. At the same time all the staff in the kindergarten got ill. Two days ago all of them ate cottage cheese with sour cream. General condition of patients is of moderate severity. Temperature 38.2 oC. Heart tones: rhythmic and muted. Heart rate 95/min, arterial pressure: 160 mm/Hg. Abdomen is slightly swollen, painful. Liver +2 cm. What is the most likely diagnosis?

A. *Salmonellosis

B. Shigellosis

C. Cholera

D. Food toxic infection

E. Enterovirus infection

46. A 12 year old female complained of right subcostal aching pain, nausea, and decreased appetite. She fell ill 2 months after appendectomy when jaundice appeared. She was treated in an infectious hospital. 1 year later above mentioned symptoms developed. On exam: the subicteric sclerae, enlarged firm liver. Your preliminary diagnosis:

A. *Chronic viral hepatitis

B. Calculous cholecystitis

C. Gilbert’s disease

D. Acute viral hepatitis

E. Chronic cholangitis

47. A 10 month old boy has been ill for 5 days after consumption of unboiled milk. Body temperature is 38?39 oC, there is vomiting, liquid stool. The child is pale and inert. His tongue is covered with white deposition. Heart sounds are muffled. Abdomen is swollen, there is borborygmus in the region of umbilicus, liver is enlarged by 3 cm. Stool is liquid, dark-green, with admixtures of mucus, 5 times a day. What is the most probable diagnosis?

A. *Salmonellosis

B. Staphylococcal enteric infection

C. Escherichiosis

D. Acute shigellosis

E. Rotavirus infection

48. A 13-years-old patient was delivered to the infectious diseases department on the 7-th day of disease. He complained about great weakness, high temperature, pain in the lumbar area and leg muscles, icteritiousness, dark color of urine, headache. The acute disease started with chill, body temperature rise up to 40 oC, headache, pain in the lumbar area and crural muscles. Icterus turned up on the 4th day, nasal and scleral hemorrhages came on the 5th day. Fever has lasted for 6 days. Diuresis – 200 ml. What is the most probable diagnosis?

A. *Leptospirosis

B. Typhoid fever

C. Virus A hepatitis

D. Sepsis

E. Yersiniosis

49. A 10 year old boy suffers from chronic hepatitis B with maximal activity. What laboratory test can give the most precise characteristic of cytolysis degree?

A. *Transaminase test

B. Weltman’s coagulation test

C. Takata-Ara test

D. Prothrombin test

E. Test for whole protein

50. A 15 year old patient was admitted on the 1st day of the disease with complaints of double vision in the eyes, heavy breathing. The day before the patient ate home-made mushrooms. On objective examination: paleness, mydriatic pupils, difficult deglutition, bradycardia, constipation. What is the diagnosis?

A. *Botulism

B. Yersiniosis

C. Leptospirosis

D. Salmonellosis, gastrointestinal form

E. Lambliasis

51. A patient in grave condition was delivered to the admission ward of a hospital on the 2nd day of illness. Examination revealed body temperature of 36.1 oC, sharpened features of face, dry skin that makes a fold, aphonia, convulsive twitching of some muscle groups. Acrocyanosis is present. Heart sounds are muffled, Ps is 102 bpm, AP is 50/20 mm Hg. Abdomen is soft, drawn-in, painless. Anuria is presnt. Stool is liquid in form of rice water. What is the most probable diagnosis?

A. *Cholera

B. Acute shigellosis

C. Salmonellosis

D. Escherichiosis

E. Intestinal amebiasis

52. A 12 year old girl complains about abrupt weakness, nausea, dizziness, vision impairment. The day before she ate home-made stockfish, beef. Examination revealed skin pallor, a scratch on the left knee, dryness of mucous membranes of oral pharynx, bilateral ptosis, mydriatic pupils. The girl is unable to read a simple text (mist over the eyes). What therapy would be the most adequate in this case?

A. *Parenteral introduction of polyvalent antibotulinic serum

B. Parenteral dysintoxication

C. Parenteral introduction of antibiotics

D. Gastric lavage

E. Parenteral introduction of antitetanus serum

53. A 17 year old patient complains about general weakness, spastic pain in the lower parts of his abdomen, mainly in the left iliac area, frequent defecations up to 18 times a day, feces contain admixtures of mucus and blood. The illness began abruptly 3 days ago with chills, fever, headache. General condition is moderately severe, body temperature is 37.8 oC. Sigmoid colon is spasmed and painful. What is the most probable diagnosis?

A. *Shigellosis

B. Amebiasis

C. Nonspecific ulcerative colitis

D. Yersiniosis

E. Salmonellosis

54. The patient’s disease began acutely. The frequent watery stool developed 6 hours ago. The body’s temperature is normal. Then the vomiting was joined. On examination: his voice is hoarse; eyes are deeply sunken in the orbits. The pulse is frequent. Blood pressure is low. There is no urine. What is the preliminary diagnosis?

A. *Cholera

B. Toxic food-borne infection

C. Salmonellosis

D. Shigellosis

E. Typhoid fever

55. A 14 y.o. male fell seriously ill, he feels chill, has got a fever, body temperature raised up to 38.5oC, paroxysmal pain in the left iliac region, frequent defecation in form of fluid bloody and mucous mass. Abdomen palpation reveals painfulness in its left half, sigmoid colon is spasmed. What is the most probable diagnosis?

A. *Acute shigellosis

B. Amebiasis

C. Colibacillosis

D. Nonspecific ulcerative colitis

E. Malignant tumors of large intestine

56. The patient was admitted to the hospital on the 7-th day of the disease with complaints of high temperature, headache, pain in the muscles, especially in calf muscles. The dermal integuments and scleras are icteric. There is hemorrhagic rash on the skin. Urine is bloody. The patient went fishing two weeks ago. What is the diagnosis?

A. *Leptospirosis

B. Yersiniosis

C. Salmonellosis

D. Brucellosis

E. Trichinellosis

57. A student diagnosed with “acute shigellosis” was sent to the infectious department by a doctor of aid post. What document should be used for registration of this disease?

A. *Urgent report on infectious disease

B. Statistic coupon for registration of final diagnoses

C. Outpatient’s card

D. Inpatient’s card

E. Statistic card of the patient who left in-patient hospital

58. A 16 y.o. patient was admitted to the hospital with complaints of having skin and sclera jaundice, dark urine, single vomiting, appetite loss, body temperature rise up to 38oC for 2 days. Three weeks ago he went in for fishing and shared his dishes with friends. Objectively: the patient is flabby, t oC – 36.8 оС, skin and sclera are yellowish, liver extends from under the costal margin by 3 cm, it is sensitive; spleen isn’t palpable. Urine is dark, stool is partly acholic. What is the most probable diagnosis?

A. *Virus A hepatitis

B. Leptospirosis

C. Infectious mononucleosis

D. Hemolytic anemia

E. Intestinal yersiniosis

59. An 8 y.o. boy was ill with B hepatitis one year ago. In the last 2 months he has complaints of undue fatiguability, sleep disorder, appetite loss, nausea, especially in the mornings. Skin isn’t yellowish, liver and spleen are 1 cm below the costal margins, painless. Alanine aminotransferase activity is 2.2 mcmol/L. How can this condition be estimated?

A. *Development of chronic hepatitis

B. Recurrence of hepatitis B

C. Biliary dyskinesia

D. Residual effects of acute hepatitis B

E. Development of liver cirrhosis

60. For 3 days a 14-year-old male patient had had the body temperature increase up to 38 oC, weakness, poor appetite, nausea, a single vomiting. On the 4th day the temperature was normal, the condition improved, but the jaundice developed. Objectively: moderate ictericity of skin, +3 cm enlarged liver of elastic consistency. Ortner’s, Kehr’s and Voznesensky’s symptoms are negative. What test will verify the diagnosis?

A. *IgM Anti-HAV detection

B. Complete blood count

C. Ultrasound of the abdomen

D. Total bilirubin

E. AST activity

61. A 14-year-old male visited Tajikistan. After return, he complains of fever up to 40 oC which occurs every second day and is accompanied by chills, sweating. Hepatosplenomegaly is present. Blood test results: RBC – 3×1012/l, Нb- 80 g/l, WBC – 4×109/l, eosinophils – 1%, stab neutrophils – 5%, segmented neutrophils – 60%, lymphocytes – 24%, monocytes – 10%, ESR – 25 mm/h. What is the provisional diagnosis?

A. *Malaria

B. Infectious mononucleosis

C. Sepsis

D. Typhoid fever

E. Leptospirosis

62. A 13-year-old boy has had abdominal pain, bloating, nausea, liquid fatty gray stool with putrid smell for the last 3 years. Palpation reveals epigastric tenderness, as well as tenderness in the Desjardins’ pancreatic point, Chauffard’s triangle; there is positive MayoRobson’s sign. Failure of exocrine pancreatic function has been suspected. What is the most informative method for evaluating the state of exocrine pancreatic function?

A. *Fecal elastase-1 determination

B. Blood serum trypsin determination

C. Sonography of the pancreas

D. Blood and urine amylase determination

E. Scatological study

63. A 14-year-old patient complains of weakness, jaundice, itching skin. Disease onset was 1,5 months ago: fever up to 39 oC appeared at first, with progressive jaundice developed 2 weeks later. On hospitalisation jaundice was severely progressed. Liver cannot be palpated. Gallbladder is enlarged and painless. Blood bilirubin is 190 micromole/l (accounting for direct bilirubin). Stool is acholic. What is the most likely jaundice genesis in this patient?

A. *Mechanical jaundice

B. Hepatocellular jaundice

C. Hemolytic jaundice

D. Caroli syndrome

E. Gilbert’s syndrome

64. A 15-year-old female patient complains of frequent defecation and liquid blood-streaked stools with mucus admixtures, diffuse pain in the inferolateral abdomen, 6 kg weight loss within the previous month. Objectively: body temperature – 37.4 oC, malnutrition, and skin is pale and dry. Abdomen is soft, sigmoid is painful and spasmodic, makes a rumbling sound. Liver is dense, painful, extends 3 cm below the costal margin. What is the most likely diagnosis?

A. *Non-specific ulcerative colitis

B. Bacillary dysentery

C. Sprue

D. Intestinal enzymopathy

E. Helminthic invasion

65. A 4-month-old boy has been undergoing in-patient treatment for pneumocystic pneumonia for 4 weeks. The diagnosis has been made based on clinical signs, typical X-ray presentation, presence of severe hypoxemia, positive dynamics caused by intravenous introduction of Biseptol (Cotrimoxazole). Anamnesis states that enzymelinked immuno sorbent assay (ELISA) detected antibodies to HIV in the umbilical blood. Polymerase chain reaction (PCR) was performed on the child at the ages of 1 month and 3 months, and proviral DNA was detected in the child’s blood. Viral load and number of CD4+-lymphocytes was not measured. Make the diagnosis:

A. *HIV/AIDS

B. Infectious mononucleosis

C. Pneumonia

D. Tuberculosis

E. Adenovirus infection

66. An 15-year-old patient was admitted to a hospital with complaints of headache, weakness, high fever, sore throat. Objectively: enlargement of all groups of lymph nodes was revealed. The liver is enlarged by 3 cm, spleen – by 1 cm. In blood: leukocytosis, atypical lymphocytes – 15%. What is the most probable diagnosis?

A. *Infectious mononucleosis

B. Acute lymphoid leucosis

C. Diphtheria

D. Tonsillitis

E. Adenovirus infection

67. An 11-year-old girl has been immunized according to her age and in compliance with the calendar dates. What vaccinations should the children receive at this age?

A. *Diphtheria and tetanus

B. TB

C. Polio

D. Hepatitis B

E. Pertussis

68. Medical examination of a 17-year-old student revealed generalized lymphadenopathy mainly affecting the posterior cervical, axillary and ulnar lymph nodes. There are multiple injection marks on the elbow bend skin. The man denies taking drugs, the presence of injection marks ascribes to influenza treatment. Blood count: RBCs – 3.2×1012/l, Hb – 100 g/l, WBCs – 3.1×109/l, moderate lymphopenia. What study is required in the first place?

A. *ELISA for HIV

B. Immunogram

C. Sternal puncture

D. X-ray of lungs

E. Lymph node biopsy

 

69. A neonate is 5 days old. What vaccination dose of BCG vaccine (in mg) is necessary for vaccination of this child?

A. *0.05 mg

B. 0.025 mg

C. 0.075 mg

D. 0.1 mg

E. 0.2 mg

 

70. 15 minutes after the second vaccination with DTP vaccine a 4-month-old boy exhibited the symptoms of Quincke’s edema. What medication should be given for emergency aid?

A. *Prednisolone

B. Heparin

C. Adrenalin

D. Furosemide

E. Seduxen

71. A 1-year-old child has been taken to a pediatrician. He has no recent history of any diseases. Objective examination revealed no pathology of the internal organs. The child needs the routine immunization against the following disease:

A. *Measles, rubella, parotitis

B. Poliomyelitis

C. Diphtheria and tetanus

D. Pertussis

E. Type B hepatitis

72. A 16 year old male complained of 3 weeks history of fevers and fatigue, weight loss with no other symptoms. Physical findings: Temperature 37.6 oC, Ps – 88 b.p.m., blood pressure 115/70 mm Hg, superficial lymph nodes (occipital, submental, cervical, axillary) are enlarged, neither tender nor painful. Rubella-like rash on the trunk and extremities. Herpes simplex lesions on the lips. Candidiasis of oral cavity. What infectious disease would you suspect?

A. *HIV infection

B. Influenza

C. Rubella

D. Infectious mononucleosis

E. Tuberculosis

73. A 15 y.o. female consulted a doctor about continued fever, night sweating. She lost 7 kg within the last 3 months. She had casual sexual contacts. Objectively: enlargement of all lymph nodes, hepatolienal syndrome. Blood count: leukocytes – 2.2×109/L. What disease can be suspected?

A. *HIV-infection

B. Lymphogranulomatosis

C. Tuberculosis

D. Infectious mononucleosis

E. Chroniosepsis

74. In an inhabited locality there is an increase of diphtheria during the last 3 years with separate outbursts in families. What measure can effectively influence the epidemic process of diphtheria and reduce the morbidity rate to single cases?

A. *Immunization of the population

B. Hospitalization of patients

C. Detection of carriers

D. Early diagnostics

E. Disinfection in disease focus

75. A 25-year-old patient has been admitted to the hospital with the following problems: weakness, sweating, itching, weight loss, enlarged submandibular, cervical, axillary, inguinal lymph nodes. Objectively: hepatomegaly. Lymph node biopsy revealed giant Berezovsky-Reed-Sternberg cells, polymorphocellular granuloma composed of lymphocytes, reticular cells, neutrophils, eosinophils, fibrous tissue, plasma cells. What is the most likely diagnosis?

A. *Lymphogranulomatosis

B. Lymph node tuberculosis

C. Lymphoreticulosarcoma

D. Cancer metastases to lymph nodes

E. Macofollicular reticulosis

76. A 15-year-old patient has been admitted to the hospital with the following problems: weakness, sweating, itching, weight loss, enlarged submandibular, cervical, axillary, inguinal lymph nodes. Objectively: hepatomegaly. Lymph node biopsy revealed giant Berezovsky-Reed-Sternberg cells, polymorphocellular granuloma composed of lymphocytes, reticular cells, neutrophils, eosinophils, fibrous tissue, plasma cells. What is the most likely diagnosis?

A. *Lymphogranulomatosis

B. Lymph node tuberculosis

C. Lymphoreticulosarcoma

D. Cancer metastases to lymph nodes

E. Macrofollicular reticulosis

77. A 16-year-old female undergoing chemotherapy for her oncologic disorder has developed sore throat. Examination revealed necrotic areas on the mucosa of the pharynx and tonsils. Many of her teeth are afflicted with caries. In blood: neutrophilic granulocytes are practically absent against the background of leukopenia. Leukocytes are represented mainly by lymphocytes and monocytes. What disease can be suspected in the given case?

A. *Agranulocitar tonsillitis

B. Lacunar tonsillitis

C. Pseudomembranous (Vincent’s) tonsillitis

D. Syphilitic tonsillitis

E. Diphtheria

78. A 17-year-old female, who works as a milk and dairy inspector, after the miscarriage suffers from high fever up to 38.6oC, recurring chills, excessive sweating. Objectively: polyadenitis, pain in the lumbosacralspine, swollen left knee joint, enlarged liver and spleen. What diagnosis is most likely?

A. *Brucellosis

B. Sepsis

C. Toxoplasmosis

D. Polyarticular rheumatoid arthritis

E. Yersiniosis

79. A 5-year-old child has body temperature risen up to febrile numbers, suffers from inertness, weakness. Examination revealed hemorrhage on the skin of limbs and torso. Enlargement of cervical and axillary lymph nodes can be detected. The liver is 4 cm below the costal arch; the spleen is 6 cm below the costal arch. Blood test: erythrocytes – 2.3×1012/l, Hb – 60 g/l, platelets – 40×109/l, leukocytes – 32,8×109/l, eosinophiles – 1%, band neutrophiles – 1%, segmented neutrophiles – 12%, lymphocytes – 46%, monocytes – 1%, blasts – 40%, Duke’s bleeding time test result is 9 min. What examination is necessary to make the diagnosis?

A. *Myelogram

B. Lymph nodes biopsy

C. US of abdominal cavity

D. Detection of hepatitis markers

E. Investigation of platelets dynamic functions

 

80. A 9 year old girl with a history of intermittent wheezing for several years is brought to the pediatrician. The child has been taking no medications for some time. Physical examination reveals agitation and perioral cyanosis. Intercostal and suprasternal retractions are present. The breath sounds are quiet, and wheezing is audible bilaterally. The child is admitted to the hospital. Appropriate interventions might include all of the following EXCEPT:

A. *Prescribe nebulized cromolyn sodium

B. Prescribe intravenous aminophylline = theophylline and ethylenediamine

C. Administer supplemental oxygen

D. Prescribe intravenous corticosteroids

E. Prescribe nebulized metaproterenol = Orciprenaline: ẞ 2 selective adrenergic agonist : bronchodilator used in tx of asthma.

 

Explanation:

Question speaks of an attack which should be addressed

The above symptoms indicate for an acute attack and nebulized cromolyn sodium is used for prophylaxis and long term usage. Note: Prescribe nebulized metaproterenol

Cromolyn sodium (Cromoglicic acid or cromoglicate) is a mast cell stabilizer and is slow acting/ has longer time to onset. This drug prevents the release of inflammatory chemicals such as histamine from mast cells. It is not histamine antagonist / bronchodilator therefore NOT used in ASTHMATIC ATTACK

**All other choices BCDE could be used in asthmatic attacks in emergency situation, while A not.

Aminophylline is a drug combination that contains theophylline and ethylenediamine in a 2:1 ratio. Once in the body, theophylline is released and acts as a phosphodiesterase inhibitor, adenosine receptor blocker, and histone deacetylase activator. It is used to prevent and treat wheezing, shortness of breath, and difficulty breathing caused by asthma, chronic bronchitis, emphysema, and other lung diseases. It relaxes and opens air passages in the lungs, making it easier to breathe.

 

81. The 10 y.o. boy has complains on headache, weakness, fever 40oС, vomiting, expressed dyspnea, pale skin with flush on right cheek, lag of right hemithorax respiratory movement, dullness on percussion over low lobe of right lung, weakness of vesicular respiration in this zone. The abdomen is painless and soft at palpation. Which disease lead to these symptoms and signs?

A. *Pneumonia croupousa → new name Lobar Pneumonia

B. Intestinal infection

C. Acute appendicitis

D. Acute cholecystitis

E. Flu

Explanation:

Fever, dullness on percursion of lungs and weakness of vesicular respiration are signs of pneumonia.

Signs and symptoms of pneumonia may include:
  • Chest pain when you breathe or cough.
  • Confusion or changes in mental awareness (in adults age 65 and older)
  • Cough, which may produce phlegm.
  • Fatigue.
  • Fever, sweating and shaking chills.

*Lag mean lateness , slow ( Rt. Half of chest breathing later or slower than Lt in movement )

 

82. A 3 month old infant suffering from acute segmental pneumonia has dyspnea (respiration rate – 80 per minute), paradoxical breathing, tachycardia, total cyanosis. Respiration and pulse – ratio is 1:2. The heart dullness under normal size. Such signs characterize:

A. *Respiratory failure of III degree → RR 80/min

B. Respiratory failure of I degree → RR 30/min

C. Respiratory failure of II degree → RR 60/min

D. Myocarditis

E. Congenital heart malformation

 

Explanation:

Respiratory failure degree is identified by Respiration and pulse ratio

Normal Respiration and pulse ratio 1 : 3.5 – 4

Respiratory failure of I degree 1 : 3.0 – 3.5

Respiratory failure of II degree 1 : 2.5 – 3.0

Respiratory failure of III degree 1 : 2.0 – 2.5

 

83. The 7 m.o. infant is suffering from acute pneumonia which was complicated by cardiovascular insufficiency and respiratory failure of II degree. The accompanied diagnosis is malnutrition of II degree. Choose the best variant of therapy:

A. *Ampiox and Amicacin → ampiox (ampicillin+ oxacillin) , amicacin = aminoglycosides, Penicilins and Amoxacilins have synagistic effect

B. Macropen and Penicillin

C. Penicillin and Ampiox

D. Gentamycin and Macropen

E. Ampiox and Polymixin → Never Mix

 

Explanation:

Ampiox and Amicacin is the first line of treatment for patients with pneumonia and malnutrition

Ampiox is Used for:

Bacterial infections of the ear, nose and throat, eg otitis media, sinusitis.

Bronchitis.

Pneumonia.

Bacterial infections of the urinary tract.

Gynaecological infections, eg gonorrhoea.

Bacterial infection of the blood (septicaemia or blood poisoning).

Bacterial infections in the abdomen (peritonitis).

Bacterial infections of the stomach and intestines.

Bacterial infection of the heart valves and the lining surrounding the heart (bacterial endocarditis).

Bacterial meningitis.

Typhoid and paratyphoid fever.

More about Ampiox

Amikacin injection is used to treat certain serious infections that are caused by bacteria such as meningitis (infection of the membranes that surround the brain and spinal cord) and infections of the blood, abdomen (stomach area), lungs, skin, bones, joints, and urinary tract.

 

84. A boy of 7 y.o. had an attack of asthma and distant whistling rales after playing with a dog. In the medical history: atopic dermatitis caused by eating eggs, chicken, beef. What group of allergens is the reason of the development of bronchial astma attacks?

 

A. *Epidermal → Contact & Biological

B. Dust

C. Pollen

D. Itch mite

E. Chemical

 

85. An 18-month-old child was taken to a hospital on the 4-th day of the disease. The disease began acutely with temperature 39, weakness, cough, breathlessness. He is pale, cyanotic, has had febrile temperature for over 3 days. There are crepitate fine bubbling rales on auscultation. Percussion sound is shortened in the right infrascapular region. X-ray picture shows non-homogeneous segment infiltration 8-10 mm on the right, the intensification of lung pattern. Your diagnosis:

A. *Segmentary pneumonia

B. Grippe

C. Bronchitis

D. Bronchiolitis

E. Interstitial pneumonia

 

Explanation:

Segment infiltration together with other highlighted key points indicate Segmental pneumonia.

 

86. 7 y.o. boy with chronic sinusitis and recurrent pulmonary infections has chest X-ray demonstrating a right-sided cardiac silhouette. What is the most likely diagnosis?

A. *Kartagener syndrome

B. Cystic fibrosis (mucoviscidosis)

C. Bronchiolitis obliterans

D. Laryngotracheomalacia

E. Alpha-antitrypsin deficiency

 

Explanation:

Kartagener syndrome – right-sided cardiac silhouette (situs inversus), chronic sinusitis & recurrent pulmonary infections (bronchoectasis – permanently dilated bronchus with hypersecretion of mucus & Treatment is lobectomy.)

Kartagener’s syndrome is a rare, autosomal recessive genetic ciliary disorder comprising the triad of situs inversus, chronic sinusitis, and bronchiectasis. The basic problem lies in the defective movement of cilia, leading to recurrent chest infections, ear/nose/throat symptoms, and infertility.

Diagnosis:

Triad of symptoms:

  • situs inversus totalis
  • chronic sinusitis
  • bronchiectasis

In addition his symptoms should correspond with infertility which corroborates diagnosis of Kartagener’s syndrome

After investigation we can also conclude

  • Moderate airway obstruction
  • Resp failure type 1 (hypoxia without hypercapnia)

 

87. A 4 year old girl was playing with her toys and suddenly she got an attack of cough, dyspnea. Objectively: respiration rate – 45/min, heart rate – 130/min. Percussion revealed dullness of percutory sound on the right in the lower parts. Auscultation revealed diminished breath sounds with bronchial resonance on the right. X-ray picture showed shadowing of the lower part of lungs on the right. Blood analysis revealed no signs of inflammation. The child was diagnosed with foreign body in the right bronchus. What complication caused such clinical presentations?

A. *Atelectasis

B. Emphysema

C. Pneumothorax

D. Bronchitis

E. Pneumonia

 

Explanation:

Post-Obstructive atelectasis(lung collapse) is seen in this patient due to foreign body in the right bronchus

Atelectasis is the collapse or closure of a lung resulting in reduced or absent gas exchange. It is usually unilateral, affecting part or all of one lung. It is a condition where the alveoli are deflated down to little or no volume, as distinct from pulmonary consolidation, in which they are filled with liquid.

 

88. A 6 week old child is admitted because of tachypnea. Birth had been uneventful, although conjunctivitis developed on the third day of life and lasted for about 2 weeks. Physical examination reveals tachypnea, bilateral inspiratory crackles and single expiratory wheezing. Bilateral pneumonia is evident on chest X-ray. The child is afebrile and has no history of fever. White blood cell count is 15*109/l, with 28% of eosinophils. The most likely cause of this child’s symptoms is:

A. *Chlamydia trachomatis

B. Pneumocystis carinii

C. Mycoplasma pneumoniae

D. Visceral larva migrans

E. Varicella

 

Explanation:

Newborn infant with conjuctivitis indicates sign of any of the TOUCH syndrome.

 

89. A 6-year-old boy was brought to the emergency room with a 3-hour history of fever up to 39,5oC and sore throat. The child looks alert, anxious and has a mild inspiratory stridor. You should immediately:

A. *Prepare to establish an airway

B. Obtain an arterial blood gas and start an IV line

C. Order a chest x-ray and lateral view of the neck

D. Examine the throat and obtain a culture

E. Admit the child and place him in a mist tent

 

Explanation:

inspiratory stridor indicates upper airway obstruction.

 

90. A 13 y.o. patient was treated in dermatological hospital for atopic dermatitis exacerbation. He was discharged in the condition of clinical remission. What recommendations should the doctor give to prevent exacerbations?

A. *Use of neutral creams to protect skin

B. Frequent skin washing with detergents

C. Systematic use of local corticosteroids

D. Systematic skin disinfection

E. Avoidance of skin insolation

 

91. A 3 year old child with weight deficiency suffers from permanent moist cough. In history there are some pneumonias with obstruction. On examination: distended chest, dullness on percussion over the lower parts of lungs. On auscultation: a great number of different rales. Level of sweat chloride is 80 millimol/l. What is the most probable diagnosis?

A. *Mucoviscidosis (cystic fibrosis)

B. Bronchial asthma

C. Recurrent bronchitis

D. Bronchiectasis

E. Pulmonary hypoplasia

 

Explanation:

Normal Sweat chloride is 40 to 59 mmol/L

** normal sweat chloride values are 10-35 milliequ/L.

** Cystic fibrosis usually have a sweat chloride value > 60 milliequ/L.

 

Sweat chloride test is a common and simple test used to evaluate a patient who is suspected of having cystic fibrosis (CF), the most common lethal genetic disease affecting Caucasians.

CF is often clinically suspected when there is poor growth during infancy or recurrent serious intestinal or respiratory diseases in a toddler or young child.

The genetic defect in cystic fibrosis affects the way chloride moves in and out of cells, and sweat contains chloride in the form of sodium chloride (salt). Measurement of the chloride in sweat has been the standard method for diagnosing CF for over 40 years.

Because cystic fibrosis is so common, many states include testing for the CF gene as part of the Newborn Screen; however, sweat testing is still required to confirm the diagnosis.

 

92. A 3 y.o. girl has had a temperature rise up to 38oС, rhinitis, dry superficial cough, flabbiness, appetite loss. Palpation didn’t reveal any changes over her lungs. Percussion sound has a wooden resonance, auscultation revealed puerile breathing, no rales. In blood: leukopenia, lymphocytosis, increased ESR. What is the most probable diagnosis?

A. *Acute simple tracheitis

B. Acute obstructive bronchitis

C. Recurrent bronchitis, acute condition

D. Acute simple bronchitis

E. Bilateral microfocal pneumonia

 

93. A 5-year-old girl with the transitory immunodeficiency according to T-system has a clinical picture of a right-sided pneumonia during 2 months. How pneumonia progress can be described?

A. *Delaying

B. Recidivating

C. Chronic

D. Wavelike

E. Acute

 

Explanation:

Delaying or Prolonged.

Typical course of pneumonia lasts about 5 weeks at the max.

 

94. Examination of a 4 month old child revealed some lemon-yellow squamae with fatty crusts on the scalp. What is the most probable diagnosis?

A. *Gneiss

B. Milk crust

C. Strophulus

D. Pseudofurunculosis

E. Infantile eczema

 

Explanation:

Gneiss (Cradle cap) is a yellowish, patchy, greasy, scaly and crusty skin rash that occurs on the scalp of recently born babies.

Cradle cap is the thick, yellow-brownish spots that often appear on the scalp in babies. It is sometimes known as milk crust, because the affected skin can feel crusty, and comes off in small, dandruff-like flakes. Milk crust is a misnomer–cradle cap has nothing to do with the baby being fed on milk.

Note: milk crust is the name given to the crusts present at gneiss.

Gneiss pediatrics

strophulus is a rash in infants marked by red or sometimes whitish papules surrounded by reddish halos and popularly associated with teething distress. — called also red gum, tooth rash.

Pseudofurunculosis is a deep multiple inflammation of the merocrine sweat glands that develops in weakened infants, less than 1-4 years, with poor care, excessive sweating. Typical localization on the scalp, back, buttocks, hips. Appear painful nodes of different sizes, the skin above them is red with a bluish tinge. At first they are dense, subsequently they become soft due to purulent fusion of tissues, a fistula is formed from which creamy pus is secreted; possibly the formation of ulcers heal with scar, the development of polyadenitis, an increase in body temperature. The flow is protracted, the rash appears periodically, possibly complication with phlegmon, sepsis.

Infantile eczema. In infants, atopic dermatitis (infantile eczema) usually appears as red, itchy patches that are associated with very dry skin.

 

95. A child was taken to a hospital with focal changes in the skin folds. The child was anxious during examination, examination revealed dry skin with solitary papulous elements and ill-defined lichenification zones. Skin eruption was accompanied by strong itch. The child usually feels better in summer, his condition is getting worse in winter. The child has been artificially fed since he was 2 months old. He has a history of exudative diathesis. Grandmother by his mother’s side has bronchial asthma. What is the most likely diagnosis?

A. *Atopic dermatitis

B. Contact dermatitis

C. Seborrheal eczema

D. Strophulus

E. Urticaria

 

Exp:

Atopic dermatitis (eczema) is a condition that makes your skin red and itchy. It’s common in children but can occur at any age. Atopic dermatitis is long lasting (chronic) and tends to flare periodically. It may be accompanied by asthma or hay fever.
Symptoms: Itch; Xeroderma; Inflammation

96. In the anamnesis of a 2-year-old girl there are recurrent pneumonias with signs of obstruction. There are heterogeneous moist and dry rales, respiration is weakened. Dense, viscous secretion is difficult to hawk. There are “drumsticks”, physical retardation.What is the most probable diagnosis?

A. *Mucoviscidosis, pulmonary form

B. Recidivating bronchitis

C. Bronchial asthma

D. Congenital pulmonary polycystosis

E. Pulmonary tuberculosis

 

Exp:

** normal sweat chloride values are 10-35 milliequ/L.
** Cystic fibrosis usually have a sweat chloride value > 60 milliequ/L.

Image result for manifestation of cystic fibrosis

 

97. A 2 m.o. breast-fed child suffers from cheek skin hyperemia, sporadic papulous elements on the skin of the chest and back following the apple juice introduction. The child is restless. What is the initial pediatritian’s tactics?

A. *Clarify mother’s diet and exlude obligate allergens → child got hypersensitivity from juice … you should explain mother about child diet , and should not give any food before 4 – 6 months and should not get any allergens food.

B. Refer to prescribe dermathologist

C. Administer general ultraviolet irradiation

D. Treat with claritine

E. Apply ointment with corticosteroids to affected skin areas

 

98. An 8 y.o. boy complains of constant cough along with discharge of greenish sputum, dyspnea during physical activities. At the age of 1 year and 8 months he fell ill for the first time with bilateral pneumonia that had protracted course. Later on there were recurrences of the disease 5-6 times a year, during the remission periods there was constant productive cough. What examination results will be the most important for making a final diagnosis?

A. *Bronchography → x ray with contrast media injected through the bronchus. , pt have symptoms of bronchitis

B. Roentgenography of thorax organs

C. Bacterial inoculation of sputum

D. Bronchoscopy

E. Spirography

 

99. A 3 m.o. child fell seriously ill, body temperature rised up to 37,8oC, there is semicough. On the 3-rd day the cough grew worse, dyspnea appeared. On percussion: tympanic sound above lungs, on auscultation: a lot of fine moist and wheezing rales during expiration. What is the most probable diagnosis?

A. *Acute respiratory viral infection, bronchiolitis

B. Acute respiratory viral infection, bronchopneumonia

C. Acute respiratory viral infection, bronchitis

D. Acute respiratory viral infection, bronchitis with asthmatic component

E. Acute respiratory viral infection, focal pneumonia

 

Exp:

Bronchiolitis: is blockage of the small airway in the lungs due to a viral infection. It usually only occurs in children less than two years of age.

Symptoms may include fever, cough, runny nose, wheezing, and breathing problems. More severe cases may be associated with nasal flaring, grunting, or the skin between the ribs pulling in with breathing.

If the child has not been able to feed properly, signs of dehydration may be present. respiratory syncytial virus (72% of cases) or human rhinovirus (26% of cases)

 

100. A child was born at 34 weeks of gestation in bad condition. The cardinal symptoms show respiratoty disorders: sound prolonged expiration, additional muscles taking part in breathing, crepitation rales on the background of the rough breath sounds. Assesment according to Silverman’s scale was 0, in 3 hours- 6 with presence of clinical data. What diagnostic method can determine pneumopathy’s type in the child?

A. *Chest X-ray

B. Blood test

C. Blood gases

D. Proteinogram

E. Immunologic investigation

101. A 7-year-old child complains of itchi-ng, papular erythematous rash, dry skin. Objectively: there is lichenification in the popliteal fossae and antecubital spaces. What immunologic indicator if found in the blood serum will verify the diagnosis (atopic dermatitis)?

A. *Total IgE

B. Secretory IgA

C. IgM

D. IgG

E. IgD

102. A 7-year-old patient presents with body temperature rise up to 39o C , dry cough, pain in the lateral abdomen. Objectively: there is cyanosis of the nasolabial triangle, inspiratory dyspnea with accessory muscle recruitment. Percussion reveals pulmonary dullness; among auscultation findings there are diminished breath sounds, crepitant rales. Respiratory rate is of 50/min, HR- 120/min. Evaluate the grade of respiratory failure in the patient:

A. *II

B. I

C. III

D. IV

E. 0

103. It has been suspected that a newborn has congenital diaphragmatic hernia (asphyctic incarceration). What study will allow to confirm the diagnosis?

A. *Plain radiography of the chest cavity

B. Plain abdominal radiography

C. Irrigography

D. Pneumoirrigoscopy

E. Fibroesophagogastroduodenoscopy

104. A 9-month-old child presents with fever, cough, dyspnea. The symptoms appeared 5 days ago after a contact with a person having URTI. Objectively: the child is in grave condition. Temperature of 38oC , cyanosis of nasolabial triangle is present. Respiration rate – 54/min, nasal flaring while breathing. There was percussion dullness on the right below the scapula angle, and tympanic sound over the rest of lungs. Auscultation revealed bilateral fine moist rales (crackles) predominating on the right. What is the most likely diagnosis?

A. *Acute pneumonia → complication from URTI or ARVI , from percussion we can determined that is focal pneumonia

B. URTI → Note; he have it already but then complicated.

C. Acute laryngotracheitis → (Croup) stridor & cough.

D. Acute bronchitis

E. Acute bronchiolitis

 

Exp:

DIAGNOSTIC CRITERIA OF PNEUMONIA
Clinical → Increasing of body temperature above 38°C, hyperthermia during 3-5 days and more; dry cough
at first, then moist; signs of intoxication and respiratory failure; at palpation increased voice fremitus, at
percussion over the affected area of lung a shortened tympanitis, at auscultation – hard breathing, first dry,
then moist sonorous small and medium bubble wheezing, possible crepitation over the size of lesions,
enforced bronchophonia.

X-Ray → Infiltrative shadows in the form of foci of different size and intensity, darkening of one or
several segments, lobe or several foci.

105. A 3-month-old girl has rhinitis, dyspnea, dry cough. She has been sick for 2 days. Objectively: pale skin, acrocyanosis, hypopnoe; breathing rate is 80/min; over the whole pulmonary surface there is vesiculotympanitic (bandbox) resonance observed with numerous bubbling rales (crackles). The most likely diagnosis is:

A. *Acute bronchiolitis

B. Pneumonia

C. Mucoviscidosis

D. Foreign body in airways

E. Acute bronchitis

 

106. When playing in a kindergarten a 3-year-old child sudenly developed dyspnea, paroxysmal compulsive dry cough. The face is cyanotic, the eyes are tearful. Vomiting occurred several times. Breathing is weakened over the whole right side of the chest. The provisional diagnosis is:

A. *Foreign body

B. Obstructive bronchitis

C. Bronchial asthma

D. Hysteria fit

E. Stenosing laryngotracheitis

107. A 2-year-old girl has a medical history of recurrent obstructive pneumonia. In the lungs various moist and dry crackles can be auscultated, breath sounds are dimini-shed. Sputum is thick, viscous and difficult to expectorate. Drumstick fingers and physi-cal developmental retardation are observed. What preliminary diagnosis can be made?

A. *Pulmonary mucoviscidosis

B. Recurrent bronchitis

C. Bronchial asthma

D. Congenital polycystic lungs

E. Pulmonary tuberculosis

108. A 9-month-old child presents with fever, cough, dyspnea. The symptoms appeared 5 days ago after a contact with a person having URTI. Objectively: the child is in grave condition. Temperature of 38oC , cyanosis of nasolabial triangle is present. Respiration rate – 54/min, nasal flaring while breathing. There was percussion dullness on the right below the scapula angle, and tympanic sound over the rest of lungs. Auscultation revealed bilateral fine moist rales (crackles) predominating on the right. What is the most likely diagnosis?

A. *Acute pneumonia → complication from URTI or ARVI , from percussion we can determined that is focal pneumonia

B. URTI → Note; he have it already but then complicated.

C. Acute laryngotracheitis → (Croup) stridor & cough.

D. Acute bronchitis

E. Acute bronchiolitis

 

Exp:

DIAGNOSTIC CRITERIA OF PNEUMONIA
Clinical → Increasing of body temperature above 38°C, hyperthermia during 3-5 days and more; dry cough
at first, then moist; signs of intoxication and respiratory failure; at palpation increased voice fremitus, at
percussion over the affected area of lung a shortened tympanitis, at auscultation – hard breathing, first dry,
then moist sonorous small and medium bubble wheezing, possible crepitation over the size of lesions,
enforced bronchophonia.

X-Ray → Infiltrative shadows in the form of foci of different size and intensity, darkening of one or
several segments, lobe or several foci.

 

109. A 10-year-old patient has a history of mild bronchial asthma. During a regular check-up the patient should be recommended:

A. *To avoid allergenic food

B. To avoid body tempering procedures

C. To avoid sports

D. To avoid spa treatment

E. To avoid going to the seaside

110. A 3-month-old girl has rhinitis, dyspnea, dry cough. She has been sick for 2 days. Objectively: pale skin, acrocyanosis, hypopnoe; breathing rate is 80/min.; over the whole pulmonary surface there is vesiculotympanitic (bandbox) resonance observed with numerous bubbling crackles. The most likely diagnosis is:

A. *Acute bronchiolitis

B. Pneumonia

C. Mucoviscidosis

D. Foreign body in airways

E. Acute bronchitis

111. An 8-year-old boy has a 2-year history of blotchy itchy rash appearing after eating ci-trus fruit. The first eruption occurred at the age of 6 months after the introduction of jui-ces to the baby’s diet. Father has a history of bronchial asthma, mother – that of allergic rhi-nitis. What is the most likely diagnosis?

A. *Atopic dermatitis

B. Psoriasis

C. Pityriasis Rosea

D. Urticaria

E. Quincke’s edema

112. A 4-month-old boy has been undergoi-ng in-patient treatment for pneumocystic pneumonia for 4 weeks. The diagnosis has been made based on clinical signs, typi-cal X-ray presentation, presence of severe hypoxemia, positive dynamics caused by intravenous introduction of Biseptol (Co-trimoxazole). Anamnesis states that enzyme-linked immuno sorbent assay (ELISA) detected antibodies to HIV in the umbilical blood. Polymerase chain reaction (PCR) was performed on the child at the ages of 1 month and 3 months, and proviral DNA was detected in the child’s blood. Viral load and number of CD4+-lymphocytes was not measured. Make the diagnosis:

A. *HIV/AIDS

B. Infectious mononucleosis

C. Pneumonia

D. Tuberculosis

E. Adenovirus infection

113. A 9-month-old child presents with fever, cough, and dyspnea. The symptoms appeared 5 days ago after a contact with a person suffering from URTI. Objectively: the child is in grave condition. Temperature of 38oC, cyanosis of nasolabial triangle is present. Respiration rate – 54/min., nasal flaring during breathing. There was percussion dullness on the right below the scapula angle, and tympanic sound over the rest of lungs. Auscultation revealed bilateral fine moist crackles predominating on the right. What is the most likely diagnosis?

A. *Acute pneumonia

B. URTI

C. Acute laryngotracheitis

D. Acute bronchitis

E. Acute bronchiolitis

114. A 3-month-old girl has rhinitis, dyspnea, dry cough. She has been sick for 2 days. Objectively: pale skin, acrocyanosis, hypopnoe; breathing rate is 80/min.; over the whole pulmonary surface there is vesiculotympanitic (bandbox) resonance observed with numerous bubbling crackles. The most likely diagnosis is:

A. *Acute bronchiolitis

B. Pneumonia

C. Mucoviscidosis

D. Foreign body in airways

E. Acute bronchitis

115. A 4-month-old boy has been undergoing in-patient treatment for pneumocystic pneumonia for 4 weeks. The diagnosis has been made based on clinical signs, typical X-ray presentation, presence of severe hypoxemia, positive dynamics caused by intravenous introduction of Biseptol (Cotrimoxazole). Anamnesis states that enzymelinked immuno sorbent assay (ELISA) detected antibodies to HIV in the umbilical blood. Polymerase chain reaction (PCR) was performed on the child at the ages of 1 month and 3 months, and proviral DNA was detected in the child’s blood. Viral load and number of CD4+-lymphocytes was not measured. Make the diagnosis:

A. *HIV/AIDS

B. Infectious mononucleosis

C. Pneumonia

D. Tuberculosis

E. Adenovirus infection

116. An 15-year-old patient was admitted to a hospital with complaints of headache, weakness, high fever, sore throat. Objectively: enlargement of all groups of lymph nodes was revealed. The liver is enlarged by 3 cm, spleen – by 1 cm. In blood: leukocytosis, atypical lymphocytes – 15%. What is the most probable diagnosis?

A. *Infectious mononucleosis

B. Acute lymphoid leucosis

C. Diphtheria

D. Tonsillitis

E. Adenovirus infection

117. A 4-year-old boy was vaccinated in violation of his vaccination schedule. There are complaints of pain in the throat during swallowing, headache, inertness, fever. Objectively: the child is pale; anterior cervical lymph nodes are enlarged; tonsils are swollen, with cyanotic hyperemy, and covered with greywhite coating that cannot be removed; if it is forcibly removed, tonsils bleed. The most likely diagnosis is:

A. *Oropharyngeal diphtheria

B. Lacunar tonsillitis

C. Pseudomembranous (Vincent’s) tonsillitis

D. Infectious mononucleosis

E. Follicular tonsillitis

118. A 9-year-old patient has measles. On the 6th day after the rash appeared, the boy developed a condition manifested by dyspnea, barking cough, stenotic respiration. Objectively: the rash on the face, neck and trunk turned brown. There is a brownish desquamation. Respiratory rate is 22/min. What complication should be diagnosed?

A. *Laryngotracheitis

B. Bronchitis

C. Pneumonia

D. Pharyngitis

E. Quinsy

119. On the second day of the disease a 12- year-old male patient complains of high-grade fever, headache in the region of forehead and superciliary arches, and during eye movement; aching muscles and joints. Objectively: body temperature is 39o C. Face is hyperemic, sclerae are injected. The mucous membrane of the soft palate and posterior pharyngeal wall is bright hyperemic and has petechial hemorrhages. What changes in the hemogram are typical for this disease?

A. *Leukopenia

B. Leukocytosis

C. Neutrocytosis

D. Anemia

E. Accelerated ESR

120. A 16-year-old male patient consulted a doctor abut sore throat, fever up to 38.2oC. A week before, the patient had quinsy, didn’t follow medical recommendations. On examination, the patient had forced position of his head, trismus of chewing muscles. Left peritonsillar region is markedly hyperemic, swollen. What is the provisional diagnosis?

A. *Left-sided peritonsillar abscess

B. Meningitis

C. Phlegmonous tonsillitis

D. Pharyngeal diphtheria

E. Tonsil tumor

121. Medical examination of a 17-year-old student revealed generalized lymphadenopathy mainly affecting the posterior cervical, axillary and ulnar lymph nodes. There are multiple injection marks on the elbow bend skin. The man denies taking drugs, the presence of injection marks ascribes to influenza treatment. Blood count: RBCs – 3.2×1012/l, Hb – 100 g/l, WBCs – 3.1×109/l, moderate lymphopenia. What study is required in the first place?

A. *ELISA for HIV

B. Immunogram

C. Sternal puncture

D. X-ray of lungs

E. Lymph node biopsy

 

122. A patient, aged 16, complains of headache, mainly in the frontal and temporal areas, superciliary arch, appearing of vomiting at the peak of headache, pain during the eyeballs movement, joint’s pain. On examination: excited, to 39 oC, Ps 110/min. Tonic and clonus cramps. Uncertain meningeal signs. What is the most likely diagnosis?

A. *Influenza with cerebral edema manifestations

B. Influenza, typical disease duration

C. Respiratory syncytial virus

D. Parainfluenza

E. Adenovirus infection

123. A child is 2 years old. The child complains of hoarse voice, dyspnea with obstructed inspiration. The disease started 3 days ago from dry cough and nose stuffiness. Objectively: general condition is unbalanced, stridor is present. The child’s skin is pale. Body temperature is 37.7 oC. The palatine arches are hyperemic. There is no deposit. Heart sounds are rhythmic. Auscultation of lungs reveals rough breathing sounds, crepitation is absent. Parainfluenza virus has been detected in nasopharynx lavage. What is the most likely diagnosis?

A. *Acute laryngotracheitis

B. Epiglottitis

C. Foreign body

D. Diphtheria

E. Laryngospasm

124. On the 21 day after appearance of vesiculous chickenpox rash a 7-year-old child developed ataxia, nystagmus, intention tremor, muscle hypotonia. Liquor analysis shows a low-grade lymphocytic pleocytosis, slightly increased protein rate. What complication is it?

A. *Encephalitis

B. Purulent meningitis

C. Pneumonitis

D. Acute nephritis

E. Postherpetic neuralgia

125. A child is 9 months old. The patient’s body temperature is 36.7 oC, the skin is pale, humid, there is pain in leg muscles. There is no extremities mobility, sensitivity is present. The child has been diagnosed with poliomyelitis. The causative agent of this disease relates to the following family:

A. *Picornavirus

B. Paramyxovirus

C. Toxovirus

D. Adenovirus

E. Rotavirus

126. A 5-year-old child developed an acute disease starting from body temperature rise up to 38.5oC, running nose, cough and conjunctivitis. On the 4th day the child presented with maculo-papular rash on face. Body temparature rose again up to 39.2 oC. Over the next few days the rash spread over the whole body and extremities. Mucous membrane of palate was hyperemic; there was whitish deposition on cheek mucous membrane next to molars. What is your provisional diagnosis?

A. *Measles

B. Acute viral respiratory infection

C. Yersinia

D. Enterovirus diseases

E. Rubella

127. On the next day after being taken by influenza a 16-year-old male presented with intensified headache, dizziness, nausea. Objectively: the patient is conscious, psychomotor excitement is present; there is general hyperesthesia, moderate meningeal syndrome, nystagmus. Tendon reflexes are higher on the right, right extremities display muscle weakness, right-sided pathological Babinski’s sign is present. Liquor is transparent, pressure is 220 mm of water column; cytosis is 46/3 with prevailing lymphocytes. What is the most likely diagnosis?

A. *Influenzal meningoencephalitis

B. Bacterial meningoencephalitis

C. Subarachnoidal hemorrhage

D. Parenchymatous subarachnoidal hemorrhage

E. Ischemic stroke

128. A 16 year old male complained of 3 weeks history of fevers and fatigue, weight loss with no other symptoms. Physical findings: Temperature 37.6 oC, Ps – 88 b.p.m., blood pressure 115/70 mm Hg, superficial lymph nodes (occipital, submental, cervical, axillary) are enlarged, neither tender nor painful. Rubella-like rash on the trunk and extremities. Herpes simplex lesions on the lips. Candidiasis of oral cavity. What infectious disease would you suspect?

A. *HIV infection

B. Influenza

C. Rubella

D. Infectious mononucleosis

E. Tuberculosis

129. A 14 year old patient complains about general weakness, dizziness, body temperature rise up to 37.5 oC, sore throat, neck edema, and enlargement of submaxillary lymph nodes. Objectively: mucous membrane of oropharynx is edematic and cyanotic, tonsils are enlarged and covered with films that spread beyond the tonsils and cannot be easily removed. What is the leading mechanism of this illness’ development?

A. *Action of bacterial exotoxin

B. Action of bacterial endotoxin

C. Allergic

D. Accumulation of suboxidated products

E. Bacteremia

130. A 7 y.o. girl fell ill abruptly: fever, headache, severe sore throat, vomiting. Minute bright red rash appear in her reddened skin in 3 hours. It is more intensive in axillae and groin. Mucous membrane of oropharynx is hyperemic. Greyish patches is on the tonsills. Submaxillary lymph nodes are enlarged and painful. What is your diagnosis?

A. *Scarlet fever

B. Measles

C. Rubella

D. Pseudotuberculosis

E. Enterovirus infection

 

See 168

 

131. A 12 y.o. male complains of acute throat pain, increasing upon swallowing during 3 days. Body temperature 38.3oC, neck lymph nodules are slightly enlarged and painful. Pharyngoscopically – tonsilar hyperemia, enlargement and edema, tonsils are covered by round yellow fibrinous patches around crypts openings. Beta-haemolytic streptococcus in swab analysis. What is the diagnosis?

A. *Acute membranous tonsillitis

B. Acute follicular tonsillitis

C. Pharyngeal diphtheria

D. Infectious mononucleosis

E. Pharyngeal candidiasis

 

132. A 3 y.o. girl has had a temperature rise up to 38oС, rhinitis, dry superficial cough, flabbiness, appetite loss. Palpation didn’t reveal any changes over her lungs. Percussion sound has a wooden resonance, auscultation revealed puerile breathing, no rales. In blood: leukopenia, lymphocytosis, increased ESR. What is the most probable diagnosis?

A. *Acute simple tracheitis

B. Acute obstructive bronchitis

C. Recurrent bronchitis, acute condition

D. Acute simple bronchitis

E. Bilateral microfocal pneumonia

 

133. A 15 y.o. female consulted a doctor about continued fever, night sweating. She lost 7 kg within the last 3 months. She had casual sexual contacts. Objectively: enlargement of all lymph nodes, hepatolienal syndrome. Blood count: leukocytes – 2.2×109/L. What disease can be suspected?

A. *HIV-infection

B. Lymphogranulomatosis

C. Tuberculosis

D. Infectious mononucleosis

E. Chroniosepsis

 

134. Ambulance brought to the hospital a patient with acute respiratory viral infection. The illness began suddenly with temperature rise up to 39.9 oC. He complains of headache in frontotemporal lobes, pain in eyeballs, aching of the whole body, nose stuffiness, sore throat, dry cough. At home he had a nasal hemorrhage twice. What type of acute respiratory viral infection is it?

A. *Influenza

B. Adenovirus infection

C. Parainfluenza

D. RS-infection

E. Enterovirus infection

 

135. An infant aged 1 year on the third day of common cold at night developed inspiratory stridor, hoarse voice and barking cough. Physical examination revealed suprasternal and intercostal chest retractions. There is a bluish skin discoloration moistly seen over the upper lip. The respiratory rate is 52 per min and pulse – 122 b.p.m. The body temperature is 37.5 oC. What disease does the infant have?

A. *Acute infectious croup due to viral laryngotracheitis

B. Acute laryngitis

C. Bronchopneumonia without complications

D. Acute bronchiolitis with respiratory distress

E. Acute epiglottitis

 

136. In an inhabited locality there is an increase of diphtheria during the last 3 years with separate outbursts in families. What measure can effectively influence the epidemic process of diphtheria and reduce the morbidity rate to single cases?

A. *Immunization of the population

B. Hospitalization of patients

C. Detection of carriers

D. Early diagnostics

E. Disinfection in disease focus

137. A 3-year-old child with ARVI had been administered biseptol, paracetamol, nazoferon. On the third day of treatment the baby’s condition deteriorated: he developed sore throat, stomatitis, conjunctivitis, hypersalivation, painful dark red spots on the neck, face, chest and legs, then the spots were replaced with vesicles. Examination revealed lesions of mucous membranes around the mouth and anus. What is your provisional diagnosis?

A. *Stevens-Johnson syndrome

B. Atopic dermatitis

C. Chickenpox

D. Serum sickness

E. Bullous dermatitis

138. A 9-year-old patient has measles. On the 6th day after the rash appeared, the boy developed a condition manifested by dyspnea, barking cough, stenotic respiration. Objectively: the rash on the face, neck and torso turned brown. There is a branny desquamation. Respiratory rate is 22/min. What complication should be diagnosed?

A. *Laryngotracheitis

B. Bronchitis

C. Pneumonia

D. Pharyngitis

E. Quinsy

139. A 3-year-old girl has had an increase in body temperature up to 38, 5 oC for four days. The child refuses to eat. Over the last two days, nose and mouth breathing has become difficult. Mesopharyngoscopy reveals hyperthermia and enlargement of tonsils, as well as hyperemia and bulging of the posterior wall of the oropharynx, which significantly narrows the oropharyngeal lumen. What complication of quinsy occurred in the patient?

A. *Retropharyngeal abscess

B. Paratonsillar abscess

C. Parapharyngeal abscess

D. Phlegmon of the mouth floor

E. Laryngostenosis

140. A 7-year-old patient presents with body temperature rise up to 39 oC, dry cough, pain in the lateral abdomen. Objectively: there is cyanosis of the nasolabial triangle, inspiratory dyspnea with accessory muscle recruitment. Percussion reveals pulmonary dullness; among auscultation findings there are diminished breath sounds, crepitant rales. Respiratory rate is of 50/min, HR- 120/min. Evaluate the grade of respiratory failure in the patient:

A. *II

B. I

C. III

D. IV

E. 0

141. A 17-year-old male patient consulted a therapist about malaise, chills, runny nose, aching muscles and joints, nausea and diarrhea. The patient asks to prescribe him a lot of painkillers and sedatives (tramadol or solpadein that help the best and diazepam). Pharyngeal mucosa is pale pink, clean. Auscultation reveals vesicular breathing. Tachycardia is present. The pupils are dilated, there is sluggish response to light. There are injection marks on the forearm skin. During examination, the patient’s manner is vulgar, irritable, rude and untruthful. Make a diagnosis:

A. *Opioid addiction

B. Painkillers addiction

C. Sedative drug addiction

D. Acute respiratory disease

E. Food-borne toxic infection

142. A 9-year-old girl has been admitted to a hospital for an elevated body temperature (39.8 oC), painful dry cough, abdominal pain on the right. Examination reveals dullness on percussion on the right, diminished breath sounds, crepitus. What study is required to make a diagnosis?

A. *Radiography of the chest cavity

B. USI of the chest cavity

C. Pleural puncture

D. Bronchoscopy

E. Bronhography

143. A 14-year-old male visited Tajikistan. After return, he complains of fever up to 40 oC which occurs every second day and is accompanied by chills, sweating. Hepatosplenomegaly is present. Blood test results: RBC – 3×1012/l, Нb- 80 g/l, WBC – 4×109/l, eosinophils – 1%, stab neutrophils – 5%, segmented neutrophils – 60%, lymphocytes – 24%, monocytes – 10%, ESR – 25 mm/h. What is the provisional diagnosis?

A. *Malaria

B. Infectious mononucleosis

C. Sepsis

D. Typhoid fever

E. Leptospirosis

144. An 11-year-old boy complains of general weakness, fever up to 38.2oC, pain and swelling of the knee joints, feeling of irregular heartbeat. 3 weeks ago, the child had quinsy. Knee joints are swollen, the overlying skin and skin of the knee region is reddened, local temperature is increased, movements are limited. Heart sounds are muffled, extrasystole is present, auscultation reveals apical systolic murmur that is not conducted to the left inguinal region. ESR is 38 mm/h. CRP is 2+, antistreptolysin O titre – 400. What is the most likely diagnosis?

A. *Acute rheumatic fever

B. Vegetative dysfunction

C. Non-rheumatic carditis

D. Juvenile rheumatoid arthritis

E. Reactive arthritis

145. Against the background of angina a patient has developed pain in tubular bones. Examination revealed generalized enlargement of lymph nodes, hepatolienal syndrome, sternalgia. In blood: RBCs – 3,6×1012/l, Hb – 87 g/l, thrombocytes – 45×109/l, WBCs – 13×109/l, blasts – 87%, stab neutrophiles – 1%, segmented neutrophiles – 7%, lymphocytes – 5%, ESR – 55 mm/h. What is the most likely diagnosis?

A. *Acute leukemia

B. Erythremia

C. Chronic lymphocytic leukemia

D. Chronic myeloid leukemia

E. Multiple myeloma

146. A 5-year-old child has body temperature risen up to febrile numbers, suffers from inertness, weakness. Examination revealed hemorrhage on the skin of limbs and torso. Enlargement of cervical and axillary lymph nodes can be detected. The liver is 4 cm below the costal arch; the spleen is 6 cm below the costal arch. Blood test: erythrocytes – 2.3×1012/l, Hb – 60 g/l, platelets – 40×109/l, leukocytes – 32,8×109/l, eosinophiles – 1%, band neutrophiles – 1%, segmented neutrophiles – 12%, lymphocytes – 46%, monocytes – 1%, blasts – 40%, Duke’s bleeding time test result is 9 min. What examination is necessary to make the diagnosis?

A. *Myelogram

B. Lymph nodes biopsy

C. US of abdominal cavity

D. Detection of hepatitis markers

E. Investigation of platelets dynamic functions

147. A patient addressed a doctor with complaints of increased body temperature up to 37.8 oC and moderately sore throat for the last 3 days. Objectively: mandibular lymph nodes are enlarged up to 3 cm. Palatine tonsils are hypertrophied, covered with grey coating that spreads to the uvula and anterior pillars of the fauces. What diagnosis is most likely?

A. *Oropharyngeal diphtheria

B. Infectious mononucleosis

C. Pseudomembranous (Vincent’s) tonsillitis

D. Agranulocytosis

E. Oropharyngeal candidiasis

148. A 17-year-old female, who works as a milk and dairy inspector, after the miscarriage suffers from high fever up to 38.6oC, recurring chills, excessive sweating. Objectively: polyadenitis, pain in the lumbosacralspine, swollen left knee joint, enlarged liver and spleen. What diagnosis is most likely?

A. *Brucellosis

B. Sepsis

C. Toxoplasmosis

D. Polyarticular rheumatoid arthritis

E. Yersiniosis

149. A 16-year-old female undergoing chemotherapy for her oncologic disorder has developed sore throat. Examination revealed necrotic areas on the mucosa of the pharynx and tonsils. Many of her teeth are afflicted with caries. In blood: neutrophilic granulocytes are practically absent against the background of leukopenia. Leukocytes are represented mainly by lymphocytes and monocytes. What disease can be suspected in the given case?

A. *Agranulocitar tonsillitis

B. Lacunar tonsillitis

C. Pseudomembranous (Vincent’s) tonsillitis

D. Syphilitic tonsillitis

E. Diphtheria

150. Three weeks after a case of acute tonsillitis the patient is still weak, inert, subfebrile, his retromaxillary lymph nodes are enlarged. Tonsils are flabby, stick together with arches, there are purulent plugs in lacunae. What is the most probable diagnosis?

A. *Chronic tonsillitis

B. Chronic pharyngitis

C. Acute lacunar tonsillitis

D. Paratonsillitis

E. Tonsillar tumour

151. A 15-year-old patient has been admitted to the hospital with the following problems: weakness, sweating, itching, weight loss, enlarged submandibular, cervical, axillary, inguinal lymph nodes. Objectively: hepatomegaly. Lymph node biopsy revealed giant Berezovsky-Reed-Sternberg cells, polymorphocellular granuloma composed of lymphocytes, reticular cells, neutrophils, eosinophils, fibrous tissue, plasma cells. What is the most likely diagnosis?

A. *Lymphogranulomatosis

B. Lymph node tuberculosis

C. Lymphoreticulosarcoma

D. Cancer metastases to lymph nodes

E. Macrofollicular reticulosis

152. A 7-year-old child became ill again 2 weeks after a case of tonsillitis. There are the following complaints: temperature rise up to 38 oC, hemorrhagic rash on the extremities, enlargement of the ankle joints. Blood test: hemoglobin is 120 g/l, platelets are 170×109/l, ESR is 30 mm/hour. Urine test: proteinuria up to 0.7 g/l, cylinders – 5-6 in the field of vision, erythrocytes – 8-10 in the field of vision. What mechanism of hemorrhagic syndrome is present in the given case?

A. *Vessel wall damage caused by immune complexes

B. Platelet dysfunction

C. Suppression of hematopoietic stem cells

D. Decrease of adhesive-aggregative function of platelets

E. Vessel wall damage caused by bacteria

153. A 5-year-old girl was hospitalized with complaints of pain and swelling in the right knee joint, temperature rise up to 38, 4 oC and a rash diagnosed as erythema annulare centrifugum. The signs developed 3 days after the recovery from a case of acute respiratory disease. Name the etiotropic drug to be prescribed:

A. *Augmentin

B. Methotrexate

C. Metypred (Methylprednisolone)

D. Diclofenac sodium

E. Captopril

 

154. A 7 year old girl has mild form of varicella. Headache, weakness, vertigo, tremor of her limbs, ataxia, and then mental confusion appeared on the 5th day of illness. Meningeal signs are negative. Cerebrospinal fluid examination is normal. How can you explain these signs?

A. *Encephalitis → previous viral infection (Varicella) cause of chickenpox and herpes zoster , then appear neurological symptoms = mean the infection reach Brain , also meningeal signs -ve

B. Meningitis

C. Meningoencephalitis → same details as above just add to it meningeal signs +ve

D. Myelitis

E. Neurotoxic syndrome

 

Explanation:

The clinical manifestations of varicella-zoster virus (VZV) infections of the central nervous system (CNS) include aseptic meningitis, encephalitis, cerebral infarction associated with granulomatous vasculitis, myelitis, and multiple cranial neuropathies

 

155. A 14-year-old male has developed increasing headaches, vertigo, diplopia, paresis of the facial muscles on the right, choking during swallowing. The signs appeared on the 5th day of respiratory disorder. He was diagnosed with acute viral encephalitis. Determine the main direction of emergency therapy:

A. *Zovirax (Aciclovir)

B. Glucocorticoids

C. Ceftriaxone

D. Lasix (Furosemide)

E. Neohaemodes

156. A 17-year-old male patient complains of rash, severe pain in the subscapular region on the right. Objectively: skin in the right subscapular region is covered with linearly arranged pink-red edematous lesions that are somewhat infiltrated, and have clear boundaries. On the lesion surface there are vesicles with transparent exudate. What is the most likely diagnosis?

A. *Herpes zoster

B. Duhring dermatitis

C. Erysipelas

D. Atopic dermatitis

E. Impetigo

157. A general practitioner visited a 2-year-old child and diagnosed him with measles. The child attends a nursery, has a 5-year-old sister. What document must be filled in for the effective antiepidemic measures in the given health locality?

A. *Emergency notification on infectious disease (form № 058/o)

B. Carer’s leave certificate

C. Infant’s record (report form № 112/o)

D. House call record (form № 031/o)

E. Sick leave

158. A 3-year-old child with ARVI had been administered biseptol, paracetamol, nazoferon. On the third day of treatment the baby’s condition deteriorated: he developed sore throat, stomatitis, conjunctivitis, hypersalivation, painful dark red spots on the neck, face, chest and legs, then the spots were replaced with vesicles. Examination revealed lesions of mucous membranes around the mouth and anus. What is your provisional diagnosis?

A. *Stevens-Johnson syndrome

B. Atopic dermatitis

C. Chickenpox

D. Serum sickness

E. Bullous dermatitis

159. On the second day after preventive vaccination a 2-year-old boy presented with abdominal pain without clear localization, body temperature rose up to 38 oC. On the third day the child got red papular haemorrhagic eruption on the extensor surfaces of limbs and around the joints. Knee joints were edematic and slightly painful. Examination of other organs and systems revealed no pathological changes. What is the most likely diagnosis?

A. *Haemorrhagic vasculitis

B. Thrombocytopenic purpura

C. Meningococcemia

D. Urticaria

E. DIC syndrome

160. A 5-year-old child developed an acute disease starting from body temperature rise up to 38.5oC, running nose, cough and conjunctivitis. On the 4th day the child presented with maculo-papular rash on face. Body temparature rose again up to 39.2 oC. Over the next few days the rash spread over the whole body and extremities. Mucous membrane of palate was hyperemic; there was whitish deposition on cheek mucous membrane next to molars. What is your provisional diagnosis?

A. *Measles

B. Acute viral respiratory infection

C. Yersinia

D. Enterovirus diseases

E. Rubella

161. 2 days ago a patient presented with acute pain in the left half of chest, general weakness, fever and headache. Objectively: between the 4 and 5 rib on the left the skin is erythematous, there are multiple groups of vesicles 2-4 mm in diameter filled with transparent liquid. What diease are these symptoms typical for?

A. *Herpes zoster

B. Pemphigus

C. Herpes simplex

D. Streptococcal impetigo

E. Herpetiform Duhring’s dermatosis

162. A child is 4 years old, has been ill for 5 days. There are complaints of cough, skin rash, to 38.2 oC, face puffiness, photophobia, conjunctivitis. Objectively: there is bright, maculo-papulous, in some areas confluent rash on the face, neck, upper chest. The pharynx is hyperemic. There are seropurulent discharges from the nose. Auscultation revealed dry rales in lungs. What is the most likely diagnosis?

A. *Measles

B. Adenovirus infection

C. Scarlet fever

D. Rubella

E. Enterovirus exanthema

163. A 3 year old child has been suffering from fever, cough, coryza, conjunctivitis for 4 days. He has been taking sulfadimethoxine. Today it has fever up to 39 oC and maculopapular rash on its face. Except of rash the child’s skin has no changes. What is your diagnosis?

A. *Measles

B. Allergic rash

C. Rubella

D. Scarlet fever

E. Pseudotuberculosis

 

Explanation:

Measles

  • Maculopapular rash = raseola rash started at face and goes down = descending rash,
  • In addition to Koplik’s spot
  • 3 C’s (cough, coryza, conjunctivitis)

Rubella

  • maculopapular rash = raseola rash started at lower extremities and buttocks then goes up = Ascending rash With occipital lymph node

 

Image result for rubeola vs rubella"

 

164. A 2 year old girl has been ill for 3 days. Today she has low grade fever, severe catarrhal presentations, slight maculopapular rash on her buttocks and enlarged occipital lymph nodes. What is your diagnosis?

A. *Rubella

B. Scarlet fever

C. Measles

D. Adenovirus infection

E. Pseudotuberculosis

 

Explanation:

Rubella – also called (German measls) ascending rash , occipital LN , Forschheimer spot in 20%
of pt (palatine petechial rash) , not specific for rubella.

 

165. A 3 year old boy fell ill abruptly: fever up to 39oC, weakness, vomiting. Hemorrhagic rash of various size appeared on his lower limbs within 5 hours. Meningococcemia with infective-toxic shock of the 1 degree was diagnosed. What medications should be administered?

A. *Chloramphenicol succinate and prednisone → nitrobenzene derivate and broad-spectrum antibiotic + steroid

B. Penicillin and prednisone

C. Penicillin and immunoglobulin

D. Chloramphenicol succinate and interferon

E. Ampicillin and immunoglobulin

 

Explanation:

In case of bacteriemia and shock one should administer bacteriostatic medication not bacteriocidal to avoid de-granulation of bacteria and release of toxins into blood which could worsen the condition.

Chloramphenicol is a broad-spectrum antibiotic that was derived from the bacterium Streptomyces venezuelae and is now produced synthetically. … Chloramphenicol is bacteriostatic but may be bactericidal in high concentrations or when used against highly susceptible organisms.

Penicillins are bactericidal beta-lactam antibiotics that inhibit bacterial cell wall synthesis. A natural product, the penicillin structure has been modified to prepare a variety of semi-synthetic agents. The spectrum of antibacterial activity varies with each class of the penicillin family.

Ampicillin is a broad-spectrum, semi-synthetic, beta-lactam penicillin antibiotic with bactericidal activity. Ampicillin trihydrate inhibits bacterial cell wall synthesis by binding to penicillin binding proteins, thereby inhibiting peptidoglycan synthesis, a critical component of the bacterial cell wall.

Image result for bactericidal vs bacteriostatic"

 

166. A 4 month old child fell seriously ill: body temperature rose up to 38.5 oC, the child became inert and had a single vomiting. 10 hours later there appeared rash over the buttocks and lower limbs in form of petechiae, spots and papules. Some hemorrhagic elements have necrosis in the center. What is the most probable disease?

A. *Meningococcemia

B. Rubella

C. Influenza

D. Haemorrhagic vasculitis

E. Scarlet fever

167. A patient complained about general weakness, fever, painful rash on his trunk skin. He has been suffering from this for 3 days. Objectively: lateral surface of trunk on the left is hyperemic and edematic, there are some groups of vesicles with serous and haemorrhagic contents. What is the most probable diagnosis?

A. *Herpes zoster

B. Contact dermatitis simplex

C. Contact allergic dermatitis

D. Microbial eczema

E. Herpetiform Duhring’s dermatosis

 

168. A 7 y.o. girl fell ill abruptly: fever, headache, severe sore throat, vomiting. Minute bright red rash appear in her reddened skin in 3 hours. It is more intensive in axillae and groin. Mucous membrane of oropharynx is hyperemic. Greyish patches is on the tonsills. Submaxillary lymph nodes are enlarged and painful. What is your diagnosis?

A. *Scarlet fever

B. Measles

C. Rubella

D. Pseudotuberculosis

E. Enterovirus infection

 

Explanation:

axillae and groin = pastia lines

Image result for measles vs scarlet fever"

 

169. A 36 y.o. woman is in the 12-th week of her first pregnancy. She was treated for infertility in the past. She contacted a child who fell ill with rubella 2 days after their meeting. Woman doesn’t know if she has ever been infected with rubella. What is the adequate tactics?

A. *Monitoring of the specific IgG, IgM with the ELISA

B. Fetus wastage

C. Immunoglobulin injection

D. Cyclovin prescription

E. Interferon prescription

170. A 1.5 y.o. child fell seriously ill: chill, body temperature rise up to 40.1 oC, then rapid dropping to 36.2 oC, skin is covered with voluminous hemorrhagic rash and purple cyanotic spots. Extremities are cold, face features are sharpened. Diagnosis: meningococcosis, fulminant form, infection-toxic shock. What antibiotic must be used at the pre-admission stage?

A. *Soluble Levomycetine succinate

B. Penicillin

C. Lincomycin

D. Gentamycin

E. Sulfamonometoxin

171. A 9-year-old patient has measles. On the 6th day after the rash appeared, the boy developed a condition manifested by dyspnea, barking cough, stenotic respiration. Objectively: the rash on the face, neck and torso turned brown. There is a branny desquamation. Respiratory rate is 22/min. What complication should be diagnosed?

A. *Laryngotracheitis

B. Bronchitis

C. Pneumonia

D. Pharyngitis

E. Quinsy

 

172. A 13-year-old female patient complains of rash on the skin of her right leg, pain, weakness, body temperature rise up to 38 oC. The disease is acute. Objectively: there is an edema on the skin of her right leg in the foot area, a well-defined bright red spot in the irregular shape of flame tips, which is hot to touch. There are isolated vesicles in the focus. What is your provisional diagnosis?

 

A. *Erysipelas

B. Microbial eczema

C. Contact dermatitis

D. Toxicoderma

E. Haemorrhagic vasculitis

 

173. An 8-year-old boy has a 2-year history of blotchy itchy rash appearing after eating citrus fruit. The first eruption occurred at the age of 6 months after the introduction of juices to the baby’s diet. Father has a history of bronchial asthma, mother – that of allergic rhinitis. What is the most likely diagnosis?

 

A. *Atopic dermatitis

B. Psoriasis

C. Pityriasis Rosea

D. Urticaria

E. Quincke’s edema

 

** Advanced family history of allergies

 

174. An 8 year old child has low-grade fever, arthritis, colicky abdominal pain and a purpuric rash localized on the lower extremities. laboratory studies reveal a guaiac-positive stool, urinalysis with red blood cell (RBC) casts and mild proteinuria, and a normal platelet count. The most likely diagnosis is:

 

A. *Henoch-Schonlein’s vasculitis → called also Henoch–Scholein Purpura or vasculitis (HSP) ,or anaphylactoid purpura

B. Systemic lupus erythematosus (SLE) → butterfly rash

C. Rocky Mountain spotted fever → hemorrhagic fever/ Rickettsia rickettsii, rash appear in wrirst and ankle then palms and soles then spread centrally to the trunk and face (mostly north and south America)

D. Idiopathic thrombocytopenic purpura → ↓ platelets

E. Post-streptococcal glomerulonephritis → no sign of previous infection here, like tonsillitis. As well as no glomerulonephritis

 

Explanation:

4 Affected systems:

  • Joint Form
  • Abdominal Form
  • Cutaneous Form
  • Renal Form

**stool guaiac-positive this test to detect blood with stool

Image result for mnemonic for henoch schonlein purpura"Image result for henoch schonlein purpura how to approach"

 

 

175. The child is 11 m.o. He suffers from nervous-arthritic diathesis. The increased synthesis of what acid is pathogenic at nervous-arthritic diathesis?

A. *Uric acid

B. Acetic acid

C. Phosphoric acid

D. Hydrochloric acid

E. Sulfuric acid

 

Explanation:

Uric acid is an altered reactivity of the nervous system caused by the inheritance of the morphofunctional features of the organism that determine the breakdown of the breakdown and synthesis of purine nucleotides these people are sick with gout, obesity, nephritis, urolithiasis, diabetes, cholelithiasis, early atherosclerosis. Uric acid is responsible for this.

** Diathesis (medical) a hereditary or constitutional predisposition to a disease or other disorder

 

176. A 8-year-old boy has suffered from tonsillitis. In 2 weeks he started complaining of migratory joint pain, edema of joints, restriction of movements, fever. On examination, an acute rheumatic heart disease, activity of the III-rd degree, primary rheumocarditis, polyarthritis; acute course of disease, cardiovascular failure IIA. What medication is to be prescribed?

A. *Prednisone → steroids here due to severe carditis and HF IIA

B. Cefazolin → I class cephalosporine G+ve , +/- G-ve

C. Delagil → anti malarial drug.

D. Diprazinum → Promethazine = class I anti-Histamine

E. Erythromycin → Macrolids antibiotics Group B streptococcal infection not in acute RF caused by Group A (ẞ -hemolytics)

 

Explanation:

Question talks about possibly untreated or poorly treated tonsillitis which caused systemic autoimmune reaction as seen in rheumatism and carditis.

Prednisone has immuno-modulatory effect.

 

177. An infant was born with body mass 3 kg and body length 50 cm. Now he is 3 years old. His brother is 7 years old, suffers from rheumatic fever. Mother asked the doctor for a cardiac check up of the 3-year-old son. Where is the left relative heart border located?

A. *1 cm left from the left medioclavicular line

B. 1 cm right from the left medioclavicular line

C. Along the left medioclavicular line

D. 1 cm left from he left parasternal line

E. 1 cm right from the left parasternal line

 

Explanation:

border of relative cardiac dullness

 

178. A 14-year-old boy has rheumatism. Over the last 2 years he has had 3 rheumatic attacks. What course of rheumatism does the patient have?

A. *Prolonged

B. Acute

C. Subacute

D. Latent

E. Persistent-reccurent

179. The patient with aquired heart failure has diastolic pressure of 0 mm Hg. What heart failure does the child have?

A. *Aortal insufficiency

B. Mitral stenosis

C. Aortal stenosis

D. Mitral insufficiency

E. Rheumatism

 

Explanation:

In norm BP 120/80 due to presence of blood in systemic circulation

In Aortic insufficiency during diastole the blood in the atria flows back into the left ventricle thus nothing is available to cause pressure

 

180. A newborn infant has mild cyanosis, diaphoresis, poor peripheral pule, hepatomegaly and cardiomegaly. Respiratory rate is 60 breaths per minute, and heart rate is 230 beats per minute. The child most likely has congestive heart failure caused by:

A. *Paroxysmal atrial tachycardia

B. A ventricular septal defect and transposition of the great vessels

C. Atrial flutter and partial atrioventricular block

D. Hypoplastic left heart syndrome

E. A large atrial septal defect and valvular pulmonary stenosis

181. The girl of 11 y.o. She is ill for 1 month. She has “butterfly”-type rash on face (spots and papules), pain and swelling of small joints on arms and legs, signs of stomatitis (small-sized ulcers in mouth). CBC: Нb– 80 g/L, RBC– 2,9×10^12/L, WBC– 15×10^9/L, ESR- 40 mm/hour. Urinalysis: protein– 0,33 g/L. What is the most probable diagnosis?

A. *Systemic lupus erythematosus

B. Juvenile rheumatoid arthritis, systemic type

C. Periarteriitis nodosa

D. Acute rheumatic fever

E. Dermatomyositis

182. Examination of a 9-month-old girl revealed skin pallor, cyanosis during excitement. Percussion revealed transverse dilatation of cardiac borders. Auscultation revealed continuous systolic murmur to the left of the breastbone in the 3-4 intercostal space. This murmur is conducted above the whole cardiac region to the back. What congenital cardiac pathology can be suspected?

A. *Defect of interventricular septum

B. Defect of interatrial septum

C. Coarctation of aorta

D. Fallot’s tetrad

E. Pulmonary artery stenosis

183. A 10-year-old child has been folowed-up for the dilated cardiomyopathy. The child presents with dyspnea, cardialgia. There are dense, nonmobile edemata on the lower extremities and sacrum. Ps- 120/min. The cardiac borders are extended transversely. Heart sounds are muffled, there is blowing systolic murmur at the apex and over the xiphoid process. Liver is 3 cm enlarged, urine output is reduced. The blood total protein – 58.6 g/l. In urine: protein – 0,025 g/l, WBCs – 2-4 in the field of vision, RBCs – 2-3 in the field of vision. What is the main mechanism of edema syndrome development:

A. *Venous congestion of greater circulation

B. Venous congestion of lesser circulation

C. Peripheral circulation disorder

D. Secondary nephropathy development

E. Hypoproteinemia

184. After objective clinical examination a 12 year old child was diagnosed with mitral valve prolapse. What complementary instrumental method of examination should be applied for the diagnosis confirmation?

A. *Echocardiography

B. Roentgenography of chest

C. Phonocardiography

D. ECG

E. Veloergometry

185. A child with tetralogy of Fallot is most likely to exhibit:

A. *Increased pressure in the right ventricle

B. Increased pulmonary blood flow

C. Increased pulse pressure

D. Normal pressure gradient across the pulmonary valve

E. Normal oxygen tension (PaO2) in the left ventricle

186. A 2-months-old child after preventive vaccination had a prolonged hemorrhage from the vaccination place and due to those anintramuscular hematoma. During examination of the child a considerable rise of prothrombin consumption and a significant prolongation of the activated partial thromboplastic time were found. What is the most probable diagnosis?

A. *Hemophilia

B. Werlhof’s disease

C. Henoch-Schoenlein disease

D. Hemorrhagic disease of the neonate

E. Inborn afibrinogenemia

187. A 10 y.o. boy with hemophilia has signs of acute respiratory viral infection with fever. What of the mentioned antifebrile medications are contraindicated to this patient?

A. *Acetylsalicylic acid

B. Analgin

C. Pipolphen

D. Paracetamol

E. Panadol extra

188. A 10-year-old boy underwent treatment in cardiological department for rheumatism, I acute attack of rheumatic fever, active phase, II degree. The patient was discharged in satisfactory condition. Which drug should be chosen for prevention of rheumatism recurrence?

A. *Bicillinum-5

B. Bicillinum-1

C. Erythromycin

D. Ampicillin

E. Oxacillin

189. A 12 y.o. child with acute glomerulonephritis presented with hypertensive syndrom during first days of the disease. What is the role of angiotesin II in the pathogenesis?

A. *Intensifies production and secretion of aldosterone

B. Increases heart output

C. Infibits depressive action of prostaglandins

D. Increases erythropoetin production

E. Increases renine level

190. A 5-year-old child had an attack of palpitation with nausea, dizziness, generalized fatigue. On ECG: tachycardia with heartbeat rate of 220/min. Ventricle complexes are deformed and widened. P wave is absent. What medication is to be prescribed to provide first aid?

A. *Lydocain

B. Isoptin

C. Seduxen

D. Novocainamides

E. Strophantin

 

191. A 14 year old child suffers from vegetovascular dystonia of pubertal period. He has got sympathoadrenal atack. What medicine should be used for attack reduction?

A. *Obsidan → Propranolol hydrochloride .. non-selective β blocker used to treat high blood pressure, a number of types of irregular heart rate, thyrotoxicosis, capillary hemangiomas, performance anxiety, and essential tremors

B. No-shpa → antispasmolytic

C. Amysyl → may be they mean Amaryl Tablets (Glimepiride) hypoglycemic agent

D. Aminophylline → Bronchodilator

E. Corglicone → Corglycon Cardiac glycosides Acute & chronic heart failure (intolerance of digitalis).

 

Explanation:

sympathoadrenal crises can be called diseases of the cardiovascular system, hormonal dysfunction
caused by age-related changes in the body during adolescence or menopause, during pregnancy.
Typically, the crisis begins suddenly, very quickly “dispersed” and in seconds reaches its peak.
pressure may spike to 200/100-120 mm Hg. art., pulse up to 120 to 150 and even higher.
They mean Addisonian crisis

192. A 13 year old girl was admitted to the cardiological department because of pain in the muscles and joints. Examination of her face revealed an edematic erythema in form of butterfly in the region of nose bridge and cheeks. What is the most probable diagnosis?

A. *Systemic lupus erythematosus

B. Rheumatism

C. Dermatomyositis

D. Rheumatoid arthritis

E. Periarteritis nodosa

193. A 4 y.o. boy was admitted to the hospital with complaints of dyspnea, rapid fatigability. His anamnesis registers frequent respiratory diseases. On percussion: heart borders are dilatated to the left and upwards. On auscultation: amplification of the SII above pulmonary artery, a harsh systolodyastolic “machine” murmur is auscultated between the II and the III rib to the left of breast bone, this murmur is conducted to all other points including back. AP is 100/20 mm Hg. What is the most probable diagnosis?

A. *Opened arterial duct

B. Interventricular septal defect

C. Isolated stenosis of pulmonary arterial orifice

D. Interatrial septal defect

E. Valvar aortic stenosis

194. A 12-year-old girl applied to doctor with complaints of swelling on the front part of the neck. The doctor diagnosed hyperplasia of the thyroid gland of the second degree, euthyroidism. Ultrasound suspected autoimmune thyroiditis. Blood was taken for titre of antibodies to thyroglobulin. What titre of antibodies will be diagnostically important?

A. *1:100

B. 1:50

C. 1:150

D. 1:200

E. 1:250

195. A 14-year-old girl has been presenting with irritability and tearfulness for about a year. A year ago she was also found to have diffuse enlargement of the thyroid gland (II grade). This condition was regarded as a pubertal manifestation, the girl didn’t undergo any treatment. The girl’s irritability gradually gave place to a complete apathy. The girl got puffy face, soft tissues pastosity, bradycardia, constipations. Skin pallor and gland density progressed, the skin became of awaxen hue. What disease may be suspected?

A. *Autoimmune thyroiditis

B. Diffuse toxic goiter

C. Thyroid carcinoma

D. Subacute thyroiditis

E. Juvenile basophilism

196. A 7 y.o. child had elevation of temperature tol $40^0C$ in anamnesis. For the last 3 months he presents fusiform swelling of fingers, ankle joints and knee joint, pain in the upper part of the sternum and cervical part of the spinal column. What is the most probable diagnosis?

A. *Juvenile rheumatic arthritis

B. Rheumatism

C. Toxic synovitis

D. Septic arthritis

E. Osteoarthrits

197. A 5 y.o. child with stigmas of dysembryogenesis (small chin, thick lips, opened mouth, hyperthelorismus) has systolic murmur in the second intercostal to the right of the sternum. The murmur passes to the neck and along the sternum left edge. The pulse on the left brachial artery is weakened. BP on the right arm is 110/60 mm Hg, on the left – 100/60 mm Hg. ECG results: hypertrophy of the right ventricle. What defect is the most probable?

A. *Aortic stenosis

B. Defect of interventricular septum

C. Defect of interatrial septum

D. Coarctation of the aorta

E. Open aortic duct

198. A 13 y.o. girl complains of having temperature rises up to febrile figures for a month, joint ache, periodical skin rash. Examination revealed steady enhancing of ESR, LE-cells. What is the most probable diagnosis?

A. *Systematic lupus erythematosus

B. Juvenile rheumatoid arthritis

C. Systematic scleroderma

D. Acute lymphoblast leukosis

E. Rheumatics

199. A 6 y.o child complains of thirst, polyuria, increased appetite for 2 months with weight loss for 3 kg. There has been nocturnal enuresis during last week. On examination: hyperglycemia 14mol/L. The diagnosis is diabetis mellitus I type. What is the genesis of this disease?

A. *Autoimmune

B. Viral

C. Bacterial

D. Neurogenic

E. Virus-bacterial

200. Examination of a 12 year old child revealed diffuse thyroid enlargement of the II degree. Heart auscultation revealed dullness of heart sounds, heart rate was 64/min. The child has frequent constipations, anemia. Concentration of thyreoglobulin antibodies is increased. What disease might have caused such symptoms?

A. *Autoimmune thyroiditis

B. Diffuse toxic goiter

C. Thyroid carcinoma

D. Thyroid hyperplasia

E. Endemic goiter

201. An 8-year-old girl has been admitted to the cardiology department. Objectively: there is a skin lesion over the extensor surfaces of joints with atrophic cicatrices, depigmentation, symmetrical affection of skeletal muscles (weakness, edema, hypotrophy). What disease are these changes most typical for?

A. *Dermatomyositis

B. Systemic scleroderma

C. Nodular periarteritis

D. Systemic lupus erythematosus

E. Reiter’s disease

202. A 13 y.o. teenager who suffers from hemophilia A was taken to the hospital after a fight at school. His diagnosis is right-sided hemarthros of knee joint, retroperitoneal hematoma. What should be primarily prescribed?

A. *Fresh frozen plasma

B. Aminocapronic acid

C. Washed thrombocytes

D. Placental albumin

E. Dry plasma

203. On the second day after preventive vaccination a 2-year-old boy presented with abdominal pain without clear localization, body temperature rose up to 38^oC. On the third day the child got red papular haemorrhagic eruption on the extensor surfaces of limbs and around the joints. Knee joints were edematic and slightly painful. Examination of other organs and systems revealed no pathological changes. What is the most likely diagnosis?

A. *Haemorrhagic vasculitis

B. Thrombocytopenic purpura

C. Meningococcemia

D. Urticaria

E. DIC syndrome

204. ECG revealed the following in a 10-year-old child: sharp acceleration of the heart rate – 240/min., P wave overlaps with T wave and deforms it, moderate lengthening of PQ interval, QRS complex is without changes. What pathology does this child have?

A. *Paroxysmal atrial tachycardia

B. Ventricular hypertrophy

C. Atrial hypertrophy

D. Extrasystole

E. WPW syndrome

205. A 16-year-old girl addressed a doctor with complaints of fatigability and dizziness. On heart auscultation: systolic murmur in the II intercostal area along the breastbone edge on the left. ECG revealed signs of the right ventricular hypertrophy. X-ray revealed dilatation of the pulmonary artery trunk, enlargement of the right heart. What heart disorder is it?

A. *Defect of the interatrial septum

B. Pulmonary artery valve failure

C. Coarctation of aorta

D. Fallot’s tetrad

E. Pulmonary artery outflow stenosis

206. An 8-year-old girl periodically has sudden short-term heart pains, sensation of chest compression, epigastric pain, dizziness, vomiting. Objectively: the patient is pale, respiratory rate – 40/min., jugular pulse is present. Ps- 185/min., of poor volume. BP- 75/40 mm Hg. ECG taken during an attack shows ectopic P waves, QRS wave is not deformed. At the end of an attack a compensatory pause is observed. The most likely cause of the attack is:

A. *Paroxysmal atrial tachycardia

B. Paroxysmal ventricular tachycardia

C. Complete AV-block

D. Atrial fibrillation

E. Sinus tachycardia

207. An 8-year-old boy developed a temperature of 37,5oC two days after his recovery from the case of URTI. He complains of suffocation, heart pain. Objectively: the skin is pale, tachycardia, the I heart sound is weakened, short systolyc murmur in the 4th intercostal area near the left edge of the breastbone. What heart disorder such clincal presentation is characteristic of?

A. *Nonrheumatic myocarditis

B. Myocardiodystrophy

C. Primary rheumatic carditis

D. Cardiomyopathy

E. Fallot’s tetrology

208. A prematurely born girl is now 8 months old. She has dyspnea, tachycardia, hepatosplenomegaly, physical developmental lag, limb cyanosis. There is parasternal cardiac hump, auscultation revealed systolodiastolic murmur in the II intercostal space on the left. BP is 90/0 mm Hg. What disease can be suspected?

A. *Nonclosure of interventricular septum

B. Patent ductus arteriosus

C. Stenosis of aortal valve

D. Stenosis of pulmonary artery

E. Coarctation of aorta

209. An 11-year-old boy complains of general weakness, fever up to 38,2oC, pain and swelling of the knee joints, sensation of irregular heartbeat. 3 weeks ago, the child had a case of tonsillitis. Knee joints are swollen, the overlying skin and skin of the knee region is reddened, local temperature is increased, movements are limited. Heart sounds are muffled, extrasystole is present, auscultation reveals apical systolic murmur that is not conducted to the left inguinal region. ESR is 38 mm/hour. CRP is 2+, antistreptolysin O titre – 400. What is the most likely diagnosis?

A. *Acute rheumatic fever

B. Vegetative dysfunction

C. Reactive arthritis

D. Non-rheumatic carditis

E. Juvenile rheumatoid arthritis

210. An 8-year-old boy was brought to the admission department by his parents.Parents report that he has had pain in the right knee for thelast 9months,recently mother has noticed some limitation of motionin his right leg, and morning stiffness thatdoesn’t last till the evening.What is the most likely diagnosis?

A. *Juvenile rheumatoid arthritis

B. Rheumatism

C. Osteomyelitis of the knee joint

D. Reactive arthritis

E. Traumatic arthritis

 

211. A 13-year-old boy with hypertrophic cardiomyopathy complains of dyspnea on minimal exertion. EhoCG reveals a symmetric left ventricular hypertrophy,signs of pulmonary hypertension, dilatation of the left atrium.EF is 64%.There vealed alterations are indicative of:

A. *Diastolic heart failure

B. Systolic heart failure

C. Primary pulmonary hypertension

D. Primary arterial hypertension

E. Symptomatic arterial hypertension

 

212. A baby boy was born in time, it was his mother’s 1st pregnancy. The jaundice was revealed on the 2nd day of life, then it progressed. The adynamia, vomiting and hepatomegaly were presented. The indirect bilirubin level was 275 mcmol/L, the direct bilirubin level – 5 mcmol/L, Hb-150 g/L. Mother’s blood group – 0(I), Rh+, child’s blood group – A(II), Rh+. Make a diagnosis.

A. *Hemolytic disease of newborn (АВО incompatibility), icteric type

B. Jaundice due to conjugation disorder → need to know about the enzyme glucuronyltransferase

C. Hepatitis

D. Physiological jaundice

E. Hemolytic disease of newborn (Rh – incompatibility)

 

Explanation:

In the question ABO is not matched while Rh is matched both of them +ve ,

Icterus type = mean Juandice (pre-hepatic) indirect bilirubin will increase (unconjugated)
Image result for inheritance of abo blood group"

 

213. An 8-year-old boy fell ill acutely: he presents with fever, weakness, headache, abdominal pain, recurrent vomiting, then diarrhea and tenesmus. Stools occur 12 times daily, are scanty, contain a lot of mucus, pus, streaks of blood. His sigmoid gut is tender and hardened. What is your diagnosis?

A. *Dysentery → shiegellosis / endo + exotoxin , no invation of blood stream

B. Salmonellosis → Typhoid fever / Endotoxin only / can invade blood stream

C. Cholera → rice watery stool / dehydration , Temp. normal

D. Staphylococcal gastroenteritis → food borne intoxication, 2-8h after eating /enterotoxin of S.aureus

E. Escherichiosis → acute intestinal infection caused by E.choli , usu affected 1 year old babies.

 

Explanation:

Streaks of blood & sigmoid gut is tender and hardened indicates for Dysentery or Shigellosis

Tenesmu (want but cant) a continual or recurrent inclination to evacuate the bowels, caused by disorder of the rectum or other illness.

 

214. The child has complains of the “night” and “hungry” abdominal pains. At fibroscopy in area a bulbus ofa duodenum the ulcerrative defect of 4 mms diameter is found, the floor is obtected with a fibrin, (H.p +). Administer the optimum schemes of treatment:

A. *Omeprasole – Trichopolum – Claritromicin

B. De-nol

C. Maalox – Ranitidin

D. Vicalinum – Ranitidin

E. Trichopolum

 

Explanation:

Ned to lower acidity with Proton Pump Inhibitors (Omeprazole).

Triple therapy for H.P +

Triple therapy for H-Pylory with a PPI and Antibiotics

Quadruple Therapy

Non Bismuth quadruple therapy

 

215. A mother with an infant visited the pediatrician for expertise advice. Her baby was born with body weight 3,2 kg and body length 50 cm. He is 1 year old now. How many teeth the baby should have?

A. *8

B. 10

C. 12

D. 20

E. 6

 

Explanation:

The formula is (N-4), where n=age in months ;

1 year =12 month (12-4)=8

 

216. A mother consulted a pediatrician about her son. Her son was born with body mass of 3 kg and length of 48 cm. He’s 1 year old now. What is the required normal mass?

 

A. *10,5 kg

B. 9,0 kg

C. 11,0 kg

D. 12,0 kg

E. 15,0 kg

 

Explanation:

b.w.+600+800+800+750+700+650+600+550+500+450+400+350 = 10150 g

See weight monitoring table

 

217. 6 m.o. infant was born with body’s mass 3 kg and length 50 cm. He is given natural feeding. How many times per day the infant should be fed?

A. *5 → 6 m and above

B. 7 → first 2 months (1-2)

C. 6 → 3-5 m

D. 8 → shortly after birth

E. 4

 

Explanation:

Number of daily feedings:
First 2 months of life: 7 feedings per day every 3 hours with night break in 6 hrs.
3-5 months of life: 6 feedings per day every 3,5 hours with night break in 6,5 hrs.
shortly after birth

 

218. Infant is 6,5 months now and is given natural feeding since birth. Body mass was 3,5 kg, with length 52 cm at birth. How many times per day the supplement (up feeding) should be given?

A. *2

B. 3

C. 1

D. 0

E. 4

 

Explanation:

After 6 months: 5 feedings per day every 4 hours with night break in 8 hrs.

A greater quantity of feeding should be breast fed so 3:2

 

219. A 2 month old healthy infant with good appetite is given artificial feeding since he turned 1 month old. When is it recommended to start the corrective feeding (fruit juice)?

A. *4,0 months

B. 1,5 months

C. 2,0 months

D. 3,0 months

E. 1,0 months

 

Explanation:

** I think this wrong Q , because artificial feed should start at 6 months and above. some books said
from 4-6m .. I hope they not use this Q , anyhow keep it

 

220. A 12 year old child has the ulcer disease of stomach. What is the etiology of this disease?

A. *Intestinal bacillus

B. Helicobacter pylory

C. Salmonella

D. Lambliosis

E. Influenza

 

Explanation:

Helicobacter pylory is the main suspect but this question choose Intestinal bacillus

Bacillus species are aerobic, sporulating, rod-shaped bacteria that are ubiquitous in nature. Bacillus anthracis, the agent of anthrax, is the only obligate Bacillus pathogen in vertebrates. Bacillus larvae, B lentimorbus, B popilliae, B sphaericus, and B thuringiensis are pathogens of specific groups of insects.

 

221. A nine year old child is at a hospital with acute glomerulonephritis. Clinical and laboratory examinations show acute condition. What nutrients must not be limited during the acute period of glomerulonephritis?

A. *Carbohydrates

B. Salt

C. Liquid

D. Proteins

E. Fats

 

222. A 9-year-old girl has attacks of abdominal pain after fried food. No fever. She has pain in Ker’s point. The liver is not enlarged. Portion B [duodenal probe] – 50 ml. What is your diagnosis?

A. *Biliary tracts dyskinesia, hypotonic type

B. Hepatocirrhosis

C. Acute colitis

D. Chronic duodenum

E. Peptic ulcer

 

Explanation:

Presence of pain specifically after fried food could be due to the absence of bile released which should serve the emulsification of fat, which occurs in hypotonic form of billiary dyskinesia.

** Cera point in some questions is translated from = Kehr’s point / symptom is strengthening of pain at pressure
on the area of gall-bladder, especially on deep inhalation.

** During palpation painfulness in the place of crossing of right costal arc with the external edge of direct muscle of stomach can be observed (the Kehr’s point). By superficial and deep palpation of right hypochondrium, as a rule, painfulness, increased gall-bladder is exposed, that can be important as a symptom, and sometimes determining for the diagnosis.

Kehr’s sign is the occurrence of acute pain in the tip of the shoulder due to the presence of blood or other irritants in the peritoneal cavity when a person is lying down and the legs are elevated. Kehr’s sign in the left shoulder is considered a classic symptom of a ruptured spleen.

 

223. A 2,9-kg term male infant is born to a mother who developed polyhydramnios at 34 weeks‘ gestation. At birth, the Apgar scores were 9 and 9. The infant develops choking and cyanosis with the first feed. In addition, is unable to place a nasogastric tube. What is the most likely diagnosis?

A. *Esophageal atresia → congenital closing of esophagus.

B. Choanal atresia

C. Laryngomalacia

D. Tracheal atresia

E. Respiratory distress syndrome

 

Explanation:

Note choking & cyanosis at first feed and unable to pass nasogastric tube (Esophageal atresia) while in choanal atresia the child has cyanosis and crying while feeding and reverses on stopping feeding

Choanal atresia is a congenital disorder where the back of the nasal passage (choana) is blocked, usually by abnormal bony or soft tissue (membranous) due to failed recanalization of the nasal fossae during fetal development.

Image result for esophageal atresia"

 

224. A full term infant was born after a normal pregnancy, delivery, however, was complicated by marginal placental detachment. At 12 hours of age the child, although appearing to be in good health, passes a bloody meconium stool. For determining the cause of the bleeding, which of the following diagnostic procedures should be performed first?

A. *Barium enema

B. An Apt test

C. Gastric lavage with normal saline

D. An upper gastrointestinal series

E. Platelet count, prothrombin time, and partial thromboplastin time

 

225. A newborn aged 3 days with hyperbilirubinemia (428 mkmol/L) developed following disorders. From beginning there were severe jaundice with poor suckling, hypotomia and hypodynamia. Little bit later periodical excitation, neonatal convulsions and neonatal primitive reflexes loss are noted. Now physical examination reveals convergent squint, rotatory nystagmus and setting sun eye sign. How to explain this condition?

A. *Encephalopathy due to hyperbilirubinemia

B. Skull injury

C. Brain tumour

D. Hydrocephalus

E. Spastic cerebral palsy

 

226. A 10 month old boy has been ill for 5 days after consumption of unboiled milk. Body temperature is 38-39oC, there is vomiting, liquid stool. The child is pale and inert. His tongue is covered with white deposition. Heart sounds are muffled. Abdomen is swollen, there is borborygmus in the region of umbilicus, liver is enlarged by 3 cm. Stool is liquid, dark-green, with admixtures of mucus, 5 times a day. What is the most probable diagnosis?

A. *Salmonellosis

B. Staphylococcal enteric infection

C. Escherichiosis

D. Acute shigellosis

E. Rotaviral infection

 

Exp:

Borborygmus a rumbling or gurgling noise made by the movement of fluid and gas in the intestines.

 

227. A 3 year old child with weight deficiency suffers from permanent moist cough. In history there are some pneumonias with obstruction. On examination: distended chest, dullness on percussion over the lower parts of lungs. On auscultation: a great number of different rales. Level of sweat chloride is 80 millimol/l. What is the most probable diagnosis?

A. *Mucoviscidosis (cystic fibrosis)

B. Bronchial asthma

C. Recurrent bronchitis

D. Bronchiectasis

E. Pulmonary hypoplasia

 

Explanation:

Normal Sweat chloride is 40 to 59 mmol/L

** normal sweat chloride values are 10-35 milliequ/L.

** Cystic fibrosis usually have a sweat chloride value > 60 milliequ/L.

 

Sweat chloride test is a common and simple test used to evaluate a patient who is suspected of having cystic fibrosis (CF), the most common lethal genetic disease affecting Caucasians.

CF is often clinically suspected when there is poor growth during infancy or recurrent serious intestinal or respiratory diseases in a toddler or young child.

The genetic defect in cystic fibrosis affects the way chloride moves in and out of cells, and sweat contains chloride in the form of sodium chloride (salt). Measurement of the chloride in sweat has been the standard method for diagnosing CF for over 40 years.

Because cystic fibrosis is so common, many states include testing for the CF gene as part of the Newborn Screen; however, sweat testing is still required to confirm the diagnosis.

 

228. District pediatrician examines a healthy carried 1-month-old child. The child is breast-fed. Prophylaxis of what disease will the doctor recommend to do first?

A. *Rachitis

B. Anemia

C. Hypotrophia

D. Spasmophilia

E. Parathropy

 

229. A neonate from gestation with severe gestosis of the second half was born on the 41st week with 2400 g birth weight and 50cm long. On physical examination: skin is flaccid, subcutaneous fatty cellular tissue is thin, muscle hypotonia, new-born period reflexes are decreased. Internal organs are without pathological changes. How would you estimate this child?

A. *Term infant with pre-natal growth retardation

B. Premature infant

C. Immature infant

D. Postmature infant

E. Term infant with normal body weight

 

230. A boy, aged 9, was examined: height – 127 cm (-0,36), weight – 28,2 kg (+0,96), chest circumference – 64,9 cm (+0,66), lung vital capacity – 1520 ml (-0,16). What is the complex assessment of the child’s physical development?

A. *Harmonious

B. Disharmonious

C. Apparently disharmonious

D. Excessive

E. Below the average

 

Examination:

In normal Harmonious growth the difference in sigma deviation should not be more than 1 sigma.

 

231. A child is 7 months old. Birth weight was 3450, the child is breastfed. Supplemental feeding was introduced on time. Determine the daily protein requirements for the child:

A. *3,0 g/kg

B. 2,0 g/kg

C. 2,5 g/kg

D. 3,5 g/kg

E. 4,0 g/kg

 

Exp:

The daily requirement infants

Carbohydrate during the year 12.0 – 14.0 g

Food Group Age Breast Fed Mixed Feeding
Protein 4 months 2.0 – 2.5 g 3.5 – 4.0 g
4-9 months 3.0 – 3.5 g 3.5 – 4.0 g
9-12 months 3.0 – 3.5 g 3.5 – 4.0 g

 

Food Group Age Breast Fed Mixed Feeding
Fats up to 4 months 6.5 – 6.0 g 6.5 – 6.0 g
4-9 months 6.0 – 5.5 g 6.0 – 5.5 g
9-12 months 5.5 – 5.0 g 5.5 – 5.0 g

 

232. A 10-year-old child is sick with chronic viral hepatitis B with marked activity of the process. Total bilirubin – 70 mmol/L, direct – 26 mmol/L, indirect – 44 mmol/L. АSТ – 6,2 mmol/L, АLТ – 4,8 mmol/L. What mechanism underlies the transaminase level increase of this patient?

A. *Cytolysis of hepatocytes

B. Failure of the synthetical function of the liver

C. Hypersplenism

D. Intrahepatic cholestasis

E. Failure of bilirubin conjugation

233. On the 3rd day of life a baby presented with haemorrhagic rash, bloody vomit, black stool. Examination revealed anaemia, extended coagulation time, hypoprothrombinemia, normal thrombocyte rate. What is the optimal therapeutic tactics?

A. *Vitamin K

B. Sodium ethamsylate

C. Epsilon-aminocapronic acid

D. Fibrinogen

E. Calcium gluconate

234. A 2 month old full-term child was born with weight 3500 g and was on the mixed feeding. Current weight is 4900 g. Evaluate the current weight of the child:

A. *Corresponding to the age

B. 150 g less than necessary

C. Hypotrophy of the I grade

D. Hypotrophy of the II grade

E. Paratrophy of the I grade

235. A child is 1 year old. Ater the recent introduction of complementary feeding the child has presented with loss of appetite, diarrhea with large amounts of feces and occasional vomiting, body temperature is normal. Objectively: body weight is 7 kg, the child is very pale, there are edemata of both legs, abdomen is significantly enlarged. Coprogram shows many fatty acids and soaps. The child has been diagnosed with celiac disease and administered the gluten-free diet. What is to be excluded from the ration?

A. *Cereals – wheat and oats

B. Milk and dairy products

C. Fruit

D. Animal protein

E. High digestible carbohydrates

236. A mother of a 5 y.o. girl consulted a doctor about doughter’s involuntary urination at night, nightmares, sleep disorders, slow gaining of body weight. Objectively: malnutrition, intellectual development is good, the girl can read and explains common situations quite adultly. Her skin is very pale, liver is enlarged in size. Her mother suffers from holetithiasis. What type of diathesis is the most probable in the child’s case?

A. *Gouty diathesis

B. Urine acid diathesis

C. Exudative diathesis

D. Allergic diathesis

E. Lymphohypoplastic diathesis

237. A 10 year old girl complains about abdominal pain that is arising and getting worse after eating rough or spicy food. She complains also about sour eructation, heartburn, frequent constipations, headache, irritability. She has been suffering from this for 12 months. Objectively:the girl’s diet is adequate. Tongue is moist with white deposit at the root. Abdomen is soft, painful in its epigastric part. What study method will help to make a diagnosis?

A. *Esophagogastroduodenoscopy

B. Intragastral pH-metry

C. Fractional examination of gastric juice

D. Contrast roentgenoscopy

E. Biochemical blood analysis

238. A 10 year old boy suffers from chronic viral hepatitis type B with maximal activity. What laboratory test can give the most precise characteristic of cytolysis degree?

A. *Transaminase test

B. Weltman’s coagulation test

C. Takata-Ara test

D. Prothrombin test

E. Test for whole protein

239. A baby is 3 months old. The mother consulted a pediatrician about lack of breast milk. After several test weighings it was found that the child had to receive supplementary feeding. What is the optimal milk formula for this child?

A. *Malysh

B. Milk formula № 2

C. Milk formula № 3

D. Whole cow’s milk

E. Malutka

240. A full-term baby was born with body weight of 3200 g, body length of 50 cm, Apgar score – 8-10 points. What is the optimum time for the first breast-feeding?

A. *First 30 minutes

B. First 6 hours

C. First 24 hours

D. First 48 hours

E. After 48 hours

241. A child is 12 years old. He complains of a dull aching pain in the epigastrium and right hypochondrium, that is getting worse after taking fatty or fried food, headache, weakness, nausea, low-grade fever. Abdominal palpation reveals a marked resistance of muscles in the right hypochondrium, positive Kerr’s, Ortner’s, Murphy’s symptoms. What is the most likely diagnosis?

A. *Chronic cholecystitis

B. Acute appendicitis

C. Viral hepatitis

D. Acute gastritis

E. Acute pancreatitis

 

Explanation:

Bile helps with emulsification of fats thus the pain in a patient with an inflamed gallblader

Murphy’s sign is tested for during an abdominal examination; it is performed by asking the patient to breathe out and then gently placing the hand below the costl margin on the right side at the mid-clavicular line (the approximate location of the gallbladder). The patient is then instructed to inspire (breathe in). Normally, during inspiration, the abdominal contents are pushed downward as the diaphragm moves down (and lungs expand). If the patient stops breathing in (as the gallbladder is tender and, in moving downward, comes in contact with the examiner’s fingers) and winces with a “catch” in breath, the test is considered positive. In order for the test to be considered positive, the same maneuver must not elicit pain when performed on the left side.

Kehr’s sign is the occurrence of acute pain in the tip of the shoulder due to the presence of blood or other irritants in the peritoneal cavity when a person is lying down and the legs are elevated. Kehr’s sign in the left shoulder is considered a classic symptom of a ruptured spleen. May result from diaphragmatic or peridiaphragmatic lesions, renal calculi, splenic injury or ruptured ectopic pregnancy.

 

242. Head circumference of a 1-month-old boy with signs of excitement is 37 cm, prefontanel is 2×2 cm large. After feeding the child regurgitates small portions of milk; stool is normal in respect of its volume and composition. Muscle tonus is within norm. What is the most likely diagnosis?

A. *Pylorospasm

B. Meningitis

C. Pylorostenosis

D. Microcephaly

E. Craniostenosis

243. ?A 16-year-old patient has made an appointment with an otolaryngologist. He complains of elevated body temperature and sore throat. Disease onset was 2 days ago, after the patient ate two portions of ice-cream. Pharyngoscopy shows hyperemic mucosa of the palatine tonsils, with purulent exudate in the lacunae. Make the provisional diagnosis:

A. Pseudomembranous (Vincent’s) tonsillitis

B. Acute pharyngitis

C. Diphtheria

D. *Lacunar tonsillitis

E. Follicular tonsillitis

244. A 10-year-old boy was brought into the hospital with complaints of expiratory dyspnea, respirations are 30/min. He explains his state by a change in the weather conditions. For the last 4 years the boy has been registered for regular check-ups due to his diagnosis of third degree persistent bronchial asthma. To provide emergency aid for this child, first he needs to be given:

A. *Salbutamol or short-acting ?2-agonists

B. Claritin (Loratadine)

C. Dexamethasone

D. Adrenaline

E. Euphylline (Aminophylline)

245. A 13-year-old girl for the last two weeks has been complaining of dyspnea and shin and foot edemas that appear after a physical exertion. In the morning the edemas significantly decrease. Clinical examination revealed enlarged liver and coarse systolic murmur over the heart area. Blood test and urinalysis are without changes. What is the most likely cause of edemas in this child?

A. Angioneurotic edema

B. Hepatic cirrhosis

C. *Heart failure

D. Acute pyelonephritis

E. Nephrotic syndrome

246. A 14-year-old girl came to the general practitioner with complaints of weakness, loss of appetite, headache, rapid fatigability. Her last menstruation was profuse and lasted for 14 days after the previous delay of 2 months. Objectively: the skin is pale, heart rate is 90/min., BP is 110/70 mm Hg, Hb is 88 g/L. Rectal examination: the uterus and its appendages are without changes, no discharge from the genital tracts. What complication occurred in the patient?

A. *Posthemorrhagic anemia

B. Dysmenorrhea

C. Somatoform autonomic dysfunction of hypotonic type

D. Migraine

E. Gastritis

247. A 12-year-old female complains of body weight loss despite her increased appetite, nervousness, and tremor of the extremities. Objectively: the skin is moist; the thyroid gland is diffusely enlarged, painless, soft, and mobile. Blood test: increased level of T3, Т4, and thyroid-stimulating hormone (THS). What is the most likely diagnosis?

A. *Diffuse toxic goiter

B. Autoimmune (Hashimoto’s) thyroiditis

C. Diffuse nontoxic goiter

D. Thyroid carcinoma

E. Thyroid adenoma

248. A 2-year-old child with persisting cough and subfebrile body temperature after a case of URTI developed dyspnea, cyanosis of the nasolabial triangle, percussion dullness and weakened respiration in the lower lobe of the right lung, and a slight mediastinal displacement to the left. What pulmonary pathology is likely to cause this clinical presentation?

A. Emphysema

B. Atelectasis

C. *Pleurisy

D. Bronchitis

E. Pneumonia

249. A 7-year-old boy has severe pulmonary mucoviscidosis (cystic fibrosis). He complains of dyspnea and blood expectoration. Objectively he presents with lagging physical development, acrocyanosis, hepatomegaly, drumstick fingers, and nail plates resembling a ”clock face”. Provisional diagnosis of chronic pulmonary heart disease is made. What examination would be the most informative for diagnosis confirmation?

A. Chest X-ray

B. *Doppler echocardiography

C. Rheography of the pulmonary artery

D. Ultrasound of the liver

E. Electrocardiography

 

Explanation:

Pulmonary heart disease, also known as cor pulmonale, is the enlargement and failure of the right ventricle of the heart as a response to increased vascular resistance (such as from pulmonic stenosis) or high blood pressure in the lungs.

Doppler echocardiography is a procedure that uses Doppler ultrasonography to examine the heart. An echocardiogram uses high frequency sound waves to create an image of the heart while the use of Doppler technology allows determination of the speed and direction of blood flow by utilizing the Doppler effect

 

250. A woman with the pregnancy term of 8 weeks complains of elevated temperature up to 37.6°C, skin rash that can be characterized as macular exanthema, enlargement of posterior cervical and occipital lymph nodes, small amount of bloody discharge from the genital tracts. She was examined by the infectious diseases specialist and diagnosed with rubella. What tactics should the obstetrician-gynecologist choose?

A. Prescription of antibacterial therapy

B. Treatment of incipient abortion

C. Prescription of hemostatic therapy

D. *Abortion

E. Prescription of antiviral therapy

251. A 13-year-old girl has 30% of excessive body mass, she started to gain weight at the age of 3. She has a family history of obesity. Her height and sexual development are normal for her age. The appetite is excessive. She complains of periodical headaches. Blood pressure – 120/80 mm Hg. Subcutaneous fat is evenly distributed, she has no stretch marks. There is juvenile acne on her face. What type of obesity is it?

A. Hypothalamic obesity

B. *Alimentary constitutive obesity

C. Adrenal obesity

D. Hypothalamic syndrome of puberty

E. Hypothyroid obesity

252. At night a 2-year-old child with upper respiratory tract infection suddenly developed dyspnea with labored inspiration. Objectively the skin is pale, perioral cyanosis and slight acrocyanosis are observed. Breathing is loud, respiration rate is 32/min. Jugular, supra- and infraclavicular fossae retract during breathing. Respiration is coarse on auscultation. Heart sounds are clear and sonorous, heart rate is 120/min. What condition was complicated by the development of the upper respiratory tract infection?

A. *Stenosing laryngotracheitis

B. Bronchiolitis

C. Bronchial asthma

D. Airway foreign body

E. Obstructive bronchitis

253. A child is 1 year old. After solid food was introduced into the diet, within the last several months the child developed loss of appetite, diarrhea with large amount of feces, and occasional vomiting. Body temperature remains normal. Body weight is 7 kg. The child is very pale, has leg edemas and extremely distended abdomen. Feces analysis detects high levels of fatty acids and soaps. Diagnosis of celiac disease was made and gluten-free diet was prescribed. What should be excluded from the diet in this case?

A. Milk and dairy products

B. Easily digestible carbohydrates

C. *Cereals – wheat, oats

D. Animal protein

E. Fruits

254. A 7-year-old boy has been an inpatient for 1.5 months. He had been brought to the hospital with complaints of edemas all over his body, low urine output, and headache. Clinical urinalysis; proteins – 7.1 g/L. leukocytes – 1-2 in the vision field, erythrocytes – 3-4 in the vision field. During the course of treatment the edemas gradually dissipated, headache abated, diuresis normalized. Daily urine proteins – 3 g/L. Biochemical blood test: total protein – 43.2 g/L, urea – 5.2 mmol/L, cholesterol – 9.2 mmol/L. What glomerulonephritis syndrome is the most likely to be present in the patient?

A. Isolated urinary

B. Nephritic

C. *Nephrotic

D. Mixed

E. Hematuric

255. A 5-year-old child was brought to the ENT department by an ambulance. The child presents with cough and difficult respiration. From the patient’s history it is known that the child was playing with a toy construction set, when suddenly started coughing and developed labored breathing. Examination detects periodical cough, labored expiration, and respiratory lag in the left side of the child’s thorax. Auscultation: diminished respiration on the left. Percussion: tympanitis. X-ray shows a displacement of the mediastinal organs to the right. Make the diagnosis:

A. A foreign body in the left bronchus, complete bronchostenosis

B. A foreign body in the trachea

C. A foreign body in the right bronchus, valvular bronchostenosis

D. *A foreign body in the left bronchus, valvular bronchostenosis

E. A foreign body in the right bronchus, partial bronchostenosis

256. A 22-day-oId infant developed subcutaneous red nodes from 1.0 to 1.5 cm in size on the scalp; later the nodes suppurated. Temperature increased up to 37.7°C intoxication symptoms appeared, regional lymph nodes enlarged. Complete blood count: anemia, leukocytosis, neutrocytosis, increased ESR. What diagnosis can be made?

A. –

B. Pemphigus

C. Scalp phlegmon

D. Vesiculopustulosis

E. *Pseudofurunculosis

 

257. A 9-month-old infant presents with delayed tooth eruption and fontanel closure, weakness, and excessive sweating. What type of hypovitaminosis is the most likely in this child?

A. Hypovitaminosis A

B. Hypovitaminosis B1

C. Hypovitaminosis B6

D. *Hypovitaminosis D

E. Hypovitaminosis С

 

Explanation:

Vitamin A helps form and maintain healthy teeth, skeletal and soft tissue, mucus membranes, and skin. It is also known as retinol because it produces the pigments in the retina of the eye.

Thiamin (vitamin B1) helps the body’s cells change carbohydrates into energy.Thiamin also plays a role in muscle contraction and conduction of nerve signals. Thiamin is essential for the metabolism of pyruvate.

Vitamin B6 significant to protein, fat and carbohydrate metabolism and the creation of red blood cells and neurotransmitters

Vitamin B12 is a nutrient that helps keep the body’s nerve and blood cells healthy and helps make DNA, the genetic material in all cells.

Vitamin B12 also helps prevent a type of anemia called megaloblastic anemia that makes people tired and weak.

Vitamin D helps in building strong, healthy bones and teeth. vitamin D is to maintain normal blood levels of calcium and phosphorus.

 

258. The 5-year-old child has been ill for 2 weeks. Cough attacks developed first and were then followed by reprises. During coughing the child’s face turns red and cervical veins bulge. The cough attacks induce vomiting. X-ray shows intensified bronchial pattern. Blood test: leukocytes – 16 • 109/L, lymphocytes – 72%, erythrocyte sedimentation rate – 4 mm/hour. What is the most likely diagnosis?

A. Adenovirus infection

B. Pneumonia

C. Foreign body

D. *Pertussis

E. Obstructive bronchitis

259. A 3-month-old child with signs of rickets presents with positive Chvostek, Trousseau, and Maslov signs. One day ago the parents witnessed a cyanotic attack in their child – the child broke into a cold sweat, the eyes bulged, and respiratory arrest occurred. One minute later the child drew in a loud breath and the child’s condition normalized again. What is the cause of the described signs of the disease?

A. Increase of blood calcium levels

B. Increase of blood phosphorus levels

C. *Decrease of blood calcium levels

D. Decrease of blood phosphorus levels

E. Metabolic acidosis

260. A 3-week-old infant developed large, flaccid vesicles with purulent contents on the skin of chest and abdomen. The vesicles rupture quickly. Make the provisional diagnosis:

A. Toxic erythema

B. *Pemphigus neonatorum

C. Pseudofurunculosis

D. Vesiculopustulosis

E. Pemphigus syphiliticus

261. Disease onset was acute. A child developed general weakness, pain in the joints, and elevated temperature. Later these signs became accompanied by itching skin rash manifested as erythematous spots 2-5 mm in size. The rash gradually turned hemorrhagic. Large joints are painful and swollen; pain attacks periodically occur in the paraumbilical area; there are signs of intestinal hemorrhage. What is the most likely diagnosis?

A. Scarlet fever

B. *Hemorrhagic vasculitis (Henoch-Schonlein purpura)

C. Hemorrhagic meningoencephalitis

D. Streptococcal impetigo

E. Rheumatism

262. An 8-year-old girl complains of frequent painful urination in small amounts and urinary incontinence. The signs have been present for 2 days already. She explains her disease by overexposure to cold. Costovertebral angle tenderness is absent. Complete blood count is without pathologies. Urine test: leukocytes – 20-30 in the vision field, erythrocytes – 40-50 in the vision field, unchanged, bacteriuria. What is the most likely diagnosis?

A. Glomerulonephritis

B. Vulvitis

C. Pyelonephritis

D. Urolithiasis

E. *Cystitis

263. A 13-year-old male complains of facial edemas, headache, dizziness, low urinary output, and urine discoloration (dark red). These complaints arose after a case of acute tonsillitis. On examination there are facial edemas, the skin is pale, temperature is 37.4 degrees Celsius, heart rate is 86/min., blood pressure is 170/110 mm Hg. Heart sounds are muffled, the II heart sound is accentuated over the aorta. What eliological factor is the most likely in this case?

A. Streptococcus pyogenes

B. Streptococcus viridans

C. Staphylococcus aureus

D. *Beta-hemolytic streptococcus

E. Staphylococcus saprophyticus

264. A 16-year-old male complains of chills, rhinitis, dry cough, and fever up to 38 degrees Celsius. Examination shows him to be in a moderately severe condition; there are small pale pink non-merging spots on the skin of his back, abdomen, and extremities. Palpation reveals enlarged occipital and axillary lymph nodes. No information about vaccination history could be obtained. What is the likely etiology of this disease?

A. Mumps virus

B. Streptococcus

C. Epstein-Barr virus

D. Neisseria meningitis

E. *Rubella virus

265. A 6-month-old child on breastfeeding is hospitalized in the inpatient department. After the child recovers, the doctor recommends the mother to start introducing solid food to the child’s diet. What products should be introduced to the child’s diet first?

A. Buckwheat porridge

B. Semolina porridge

C. Fermented dairy products

D. Grated apple

E. *Vegetable puree

266. 10 hours after birth a child developed jaundice, hypotonia, hyporcflexia, and moderate hepatosplenomegaly. Feces and urine are of normal color. Umbilical cord blood bilirubin is 51 mcmol/L due to unconjugated bilirubin levels. In venous blood: erythrocytes – 3.5 • 10l2/L, Hb – 140 g/L, reticulocytes – 1.5%, bilirubin – 111 ?mol/L, conjugated – 11 mcmol/L, ALT- 40 U/L, AST- 30 U/L. Mother’s blood group is A(II) Rh (-), child’s blood group is A(II) Rh(+). What laboratory test can confirm the diagnosis?

A. Viral hepatitis markers analysis

B. *Coombs test

C. Erythrocytometry

D. Measurement of erythrocyte osmotic resistance

E. Measurement of glucose 6-phosphate dehydrogenase levels in erythrocytes

267. A 3-year-old child presents with dyspnea that abates in the sitting position, occasional loss of consciousness and seizures, delayed physical development, cyanosis, drumstick fingers. Echocardioscopy detects aortic dextraposition, ventricular septal defect, pulmonary artery stenosis, and right ventricular hypertrophy. What is the most likely diagnosis?

A. Coarctation of the aorta

B. Ventricular septal defect

C. Transposition of the great vessels

D. Acquired valvular disease

E. *Tetralogy of Fallot

 

268. A 13-year-old girl for a month has been complaining of fatigability, dull pain in her right subcostal area, abdominal distension, and constipations. Abdominal palpation reveals positive Kehr, Murphy, and Ortner signs, while Desjardins and Mayo-Robson points are painless. Total bilirubin is 14.7 ?mol/L, predominantly indirect, ALT- 20 U/L, AST- 40 U/L, amylase – 6.3 mmol/L. Echocholecystography shows practically no contraction of the gallbladder, Make the provisional diagnosis:

A. Chronic hepatitis

B. Acute pancreatitis

C. *Hypokinetic biliary dyskinesia

D. Chronic pancreatitis

E. Hyperkinetic biliary dyskinesia

 

269. A 3-year-old child has been brought to a hospital with complaints of pain in the legs, fever, and loss of appetite. Objectively: pale skin and mucosa, hemorrhagic rash. Lymph nodes are enlarged, painless, dense and elastic, not matted together. Bones, joints, and abdomen are painful. The liver and spleen are enlarged. Hemogram: Hb – 88 g/L, color index – 1.3, platelets – 80 • 109/L, leukocytes – 25.8 • 109/L, lymphoblasts – 70%, ESR- 52 mm/hour. Make the provisional diagnosis:

A. *Acute leukemia

B. Acute rheumatic fever

C. Infectious mononucleosis

D. Hemorrhagic vasculitis (Henoch-Schonlein purpura)

E. Thrombocytopenic purpura

270. A 12-year-old child complains of weakness and fever. His illness initially presented as tonsillitis one month earlier. On examination: temperature – 38.9 degrees Celsius, respirations – 24/min., pulse – 100/min., blood pressure – 100/70 mm Hg, hemorrhages on the legs, enlargement of the lymph nodes. Complete blood count shows Hb – 70 g/L, RBC- 2.2 x 1012/L, WBC- 3.0 x 109/L with 32% of blasts, 1% of eosinophiles, 3% of bands, 36% of segments, 20% of lymphocytes, and 8% of monocytes, ESR- 47 mm/hour. What is the cause of anemia in this case?

A. *Acute leukemia

B. Chronic lympholeukemia

C. Aplastic anemia

D. B12-deficient anemia

E. Chronic hemolytic anemia

 

271. A 4-year-old boy on the 3rd day of ceftriaxone treatment for acute otitis (daily dosage – 1.5 grams) developed diarrhea occurring 5-6 times per day. Feces are without mucus or blood admixtures. Temperature is 36.6 degrees Celsius. Gregersen reaction (occult blood in feces) is negative. Stool culture detected no pathogenic germs. What is the most likely cause of diarrhea in this case?

A. *Antibiotic-associated diarrhea

B. Intestinal dysbiosis

C. Bacterial overgrowth syndrome

D. Ulcerative colitis

E. Crohn’s disease (regional enteritis)

 

272. A 15-year-old female complains of fatigue, dizziness, hemorrhagic rashes on the skin. She has been presenting with these signs for a month. Blood test: erythrocytes – 1.0 x 1012/L, Hb- 37 g/L, color index – 1.1, leukocytes – 1.2 x 109/L, platelets – 42 x 109/L. What analysis would be the most advisable for diagnosis making in this case?

A. *Sternal puncture (bone marrow biopsy)

B. Splenic biopsy

C. Liver biopsy

D. Coagulation studies

E. US of the gastrointestinal tract

 

Explanation:

Low levels of all blood components is indicative for problems with blood cells production like in Aplastic anaemia so you check the bone marrow

 

273. A 12-year-old boy with hypertrophic cardiomyopathy complains of dyspnea caused by the slightest physical exertion. Echocardiography detected asymmetrical left ventricular hypertrophy, signs of pulmonary hypertension, and left ventricular dilatation, its ejection fraction is 59%. These developments are indicative of:

A. *Heart failure with preserved ejection fraction

B. Heart failure with reduced ejection fraction

C. Primary pulmonary hypertension

D. Essential hypertension

E. Symptomatic arterial hypertension

274. A 10-year-old girl exhibits high level of physical development ( + 3σ), her body length increased by 10 cm within a year (which is double the norm for her age group), the number of permanent teeth corresponds with the age norm (20), the development of her secondary sex characteristics is three years ahead of her age (Ма, Р, Ах, Menarche). Development rate ahead of her biological age can occur due to:

A. *Endocrine disorders

B. Acceleration

C. Certain components of her diet

D. Sports training

E. Deficient hygienic education

 

Explanation:

Sexual development is related to endocrine disfunction.

Male genitals
G1 Childlike penis, testicles, and scrotum.
G2 Initial increase in testicular volume (>4ml). The texture of the scrotal skin becomes reddened and thinner. The enlargement of the penis is minimal or absent.
G3 Increased length of the penis. Great enlargement of testicles and scrotum.
G4 Increased length and circumference of the penis
with great enlargement of the glans. Enlarged testicles and scrotum, and increased scrotal skin pigmentation.
G5 Full development of genital organs, with adult appearance.

Breasts (females)
M1 Childlike, with elevated papilla.
M2 Breast bud: initial increase of the mammary gland, with elevated areola and papilla, forming a small mound. Areolar diameter and texture change.
M3 Great enlargement of the breasts and areola, without contour separation.
M4 Greater enlargement of the breast and areola. A second mound is formed above the breast contour.
M5 Adult mature breasts. Recession of areola to the mound of breast tissue. Pubic hair (both sexes)
P1 No pubic hair. Vellus over the pubes in no further developed than that over the abdominal wall.
P2 There is sparse growth of long, slightly pigmented, downy hair, straight or only slightly curled, appearing chiefly at base of penis or along the labia majora).
P3 Hair is considerably darker, coarser, and more
curled, and spreads sparsely over junction of pubes.
P4 Hair is now adult in type but there is no spread to the inner thighs.
P5 Adult quantity and distribution with hair presention inner thighs.
P6 Hair spreads above the pubes.

 

275. An infant has been born at the 41st week of gestation. The pregnancy was complicated with severe gestosis of the second semester. The weight of the baby is 2400 g, the height is 50 cm. Objectively: the skin is flabby, the layer of subcutaneous fat is thin, hypomyotonia, neonatal reflexes are weak. The internal organs are without pathologic changes. This newborn can be assessed as a:

A. *Full-term infant with intrauterine growth retardation

B. Premature infant

C. Immature infant

D. Postmature infant

E. Full-term infant with normal body weight

 

Explanation:

Term newborn infant who is underweight followed with the host of symptoms indicate retardation and gestosis indicates to intrauterine affection/cause.

The average weight of a newborn is around 7 to 7 1/2 pounds (3.2 to 3.4 kg).

Most full-term healthy newborns weigh anywhere from 5 pounds 11 ounces to 8 pounds 6 ounces (2.6 to 3.8 kg)

 

276. A 16-year-old female is suspected to suffer from systemic lupus erythematosus with systemic lesions of skin, vessels, joints, serous tunics, and heart tissue that developed after photosensitization. The following is detected in blood analysis: LE cells, antibodies to native ds-DNA, isolated anti-centromere antibodies, rheumatoid factor is 1:100, Wassermann reaction is positive; circulating immune complex is 120 units. What immunological indicators are considered to be specific to this disease?

A. *ds-DNA antibodies

B. Rheumatoid factor

C. Anti-centromere antibodies

D. Immunoglobulin A

E. Increased circulating immune complex

Explanation:

ds-DNA antibodies is specific to systemic lupus erythematosus

 

277. A 13-year-old adolescent complains of facial edema, headache, dizziness, low urinary output, urine discoloration (dark red). These complaints arose after the patient had had a case of acute tonsillitis. On examination there are facial edema, the skin is pale, temperature is 37.4 degrees Celsius; heart rate is 86/min., blood pressure is 170/110 mm Hg. Heart sounds are muffled, the II heart sound is accentuated over the aorta. What etiological factor is the most likely in this case?

A. *Beta-hemolytic streptococcus

B. Staphylococcus aureus

C. Streptococcus viridans

D. Streptococcus pyogenes

E. Staphylococcus saprophyticus

 

Explanation:

Tonsillitis is most often caused by common viruses, but bacterial infections can also be the cause. The most common bacterium causing tonsillitis is Streptococcus pyogenes (group A streptococcus/Group A βhemolytic streptococcus), the bacterium that causes strep throat. Other strains of strep and other bacteria also may cause tonsillitis.

 

278. A 16-year-old adolescent complains of pain in his knee and elbow joints and body temperature up to 39.5 degrees Celsius. One week and a half earlier developed sore throat. On examination his body temperature is 38.5 degrees Celsius. Swelling of the knee and elbow joints is observed. Pulse is 106/min., rhythmic. Blood pressure is 90/60 mm Hg. Cardiac borders are unchanged, heart sounds are weakened, at the cardiac apex there is soft systolic murmur. What factor would be the most indicative of the likely disease etiology?

A. *Anti-streptolysin O

B. C-reactive protein

C. Creatine kinase

D. Rheumatoid factor

E. Seromucoid

 

Explanation:

Possible history of strep throat which could be found with Anti-streptolysin O

 

279. During examination a 4-month-old child with meningococcemia presents with acrocyanosis, cold extremities, tachypnea, and thready pulse, blood pressure of 30/0 mm Hg, anuria, and sopor. What clinical syndrome is it?

A. *Septic shock

B. Neurotoxicosis

C. Exicosis

D. Encephalic syndrome

E. Acute renal failure

 

280. A 12-year-old patient complains of severe headache, double vision, weakness, fever, irritability. Objectively: body temperature is 38.1 degrees Celsius, the patient is reluctant to contact, sensitive to stimuli. There are ptosis of the left eyelid, exotropia, anisocoria S>D, pronounced meningeal syndrome. On lumbar puncture the cerebrospinal fluid flowed out under a pressure of 300 mm Hg, the fluid is clear, slightly opalescent. 24 hours later there appeared fibrin film. Protein – 1.4 g/L, lymphocytes – 600/3 per mm3, sugar – 0.3 mmol/L. What is the provisional diagnosis?

A. *Tuberculous meningitis

B. Meningococcal meningitis

C. Lymphocytic (Armstrong’s) meningitis

D. Syphilitic meningitis

E. Mumps meningitis

 

281. The right arm of a newborn is stretched along the torso with all its joints extended; the shoulder is rotated inwards, while the forearm is pronated, the hand is in the position of palmar flexion. Spontaneous movements are absent in the shoulder and elbow joints, passive movements are painless. What is the most likely diagnosis?

A. *Duchenne-Erb palsy, superior proximal type

B. Dejerine-Klumpke palsy, inferior distal type

C. Total obstetric palsy

D. Osteomyelitis of the right humerus

E. Poliomyelitis

 

Explanation:

Erb’s palsy or ErbDuchenne palsy is a paralysis of the arm caused by injury to the upper trunk C5–C6 nerves. They form part of the brachial plexus, comprising the ventral rami of spinal nerves C5–C8 and thoracic nerve T1. These injuries arise most commonly from shoulder dystocia during a difficult birth.

Image result for duchenne-erb palsy

Image result for Total obstetric palsy

Klumpke’s paralysis is a form of paralysis involving the muscles of the forearm and hand, resulting from a brachial plexus injury in which the eighth cervical (C8) and first thoracic (T1) nerves are injured either before or after they have joined to form the lower trunk.

Image result for Dejerine-Klumpke palsy, inferior distal type

Maternal obstetric palsy or obstetric maternal palsy or obstetric maternal lumbosacral plexopathy refers to an injury to the lumbosacral plexus whose signs and symptoms become evident during labor or after childbirth.

 

282. A 1-year-old child with a case of URTI suddenly developed noisy respirations with difficult inspiration, intercostal retractions, and barking cough on the 2nd night after the disease onset. What is the most likely diagnosis?

A. *Stenosing laryngotracheobronchitis

B. Acute pulmonary inflammation

C. Bronchial asthma

D. Acute bronchitis

E. Acute bronchiolitis

 

Explanation:

Noisy respirations is due to stenosis

Croup, also known as laryngotracheobronchitis, is a type of respiratory infection that is usually caused by a virus. The infection leads to swelling inside the trachea, which interferes with normal breathing and produces the classic symptoms of “barking” cough, stridor, and a hoarse voice

 

283. A 12-year-old female has been suffering for 2 years from dull, occasionally exacerbating pain in her right subcostal area, occurring after eating high-fat foods, bitter taste in her mouth in the morning, constipations, and flatulence. Objectively she has excess weight, her body temperature is 36.9 degrees Celsius; there is a coating on the root of her tongue; the abdomen is moderately distended and painful in the area of gallbladder projection. What examination would be the most helpful for diagnosis-making?

A. *Ultrasound

B. Duodenal intubation

C. Cholecystography

D. Duodenoscopy

E. Liver scanning

284. A 17-year-old female complains of weakness, dyspnea, loss of appetite, and liquid feces. She has been suffering from this condition for 2 years. Objectively she presents with pale skin, subicteric sclera, and bright red fissured tongue. Lymph nodes are not enlarged. Pulse – 100/min. BP- 105/70 mm Hg. Liver +3 cm, the spleen cannot be palpated. Blood test: erythrocytes – 1.2х1012/L, Нb- 56 g/L, color index – 1.4, macrocytes, leukocytes – 2,5×109/L, eosinophils – 1%, juvenile – 1%, metamyelocytes – 1%, band neutrophils – 8%, segmented neutrophils – 47%, lymphocytes – 38%, monocytes – 4%, reticulocytes – 0.1%, platelets – 100×109/L, ESR- 30 mm/hour, indirect bilirubin – 26 mmol/L. What changes can be expected in the bone marrow puncture material?

A. *Prevalence of megaloblasts

B. Increased number of sideroblasts

C. Erythroid hyperplasia

D. Presence of blast cells

E. Prevalence of lymphoid tissue

 

285. A 10-year-old boy with symptoms of arthritis and myocarditis was delivered into a hospital. Based on clinical examination the preliminary diagnosis of juvenile rheumatoid arthritis was made. What symptom is the most contributive for the diagnostics of this disease?

A. *Reduced mobility of the joints in the morning

B. Regional hyperemia of the joints

C. Affection of the large joints

D. Enlarged heart

E. Increased heart rate

 

Explanation:

Morning Stiffness(Reduced mobility of the joints in the morning)

 

286. A 15-year-old adolescent complains of skin hemorrhages after minor traumas and spontaneous appearance of hemorrhages on the front of her torso and extremities. On examination: the skin is variegated (old and new hemorrhages), bleeding gums. Blood platelets – 20 x 109/L; in the bone marrow there is increased number of megakaryocytes and no platelet production. Treatment with steroid hormones was effective. What is the likely diagnosis?

A. *Idiopathic thrombocytopenic purpura

B. Hemophilia

C. Rendu-Osler-Weber disease (Hereditary hemorrhagic telangiectasia)

D. Disseminated intravascular coagulation

E. Acute vascular purpura

 

287. The mother of a 3-month-old child came to a family doctor with complaints of her child being physically underdeveloped and suffering from cough attacks and dyspnea. Anamnesis: the child is the result of the second full-term pregnancy with the risk of miscarriage (the first child died of pulmonary pathology at the age of 4 months, according to the mother). Body mass at birth is 2500 g. Cough attacks were observed from the first days of life, twice the child was treated for bronchitis. Considering the severity of the child’s condition the doctor made the referral for hospitalization. What diagnosis was most likely stated in the referral?

A. *Mucoviscidosis (Cystic fibrosis)

B. Acute obstructive bronchitis

C. Recurrent obstructive bronchitis

D. Pertussis

E. Acute obstructive pneumonia

 

Explanation:

Mucoviscidosis: Another name for cystic fibrosis (CF), the most common life-shortening autosomal recessive disease among Caucasian populations, with a frequency of 1 in 2000 to 3000 live births. The thick mucus in the lungs causes people with cystic fibrosis to get frequent lung infections.

 

288. A 10-year-old girl complains of stomachache that appears and intensifies after she eats rough or spicy food, sour eructation, heartburn, frequent constipations, headaches, irritability. She has been presenting with these signs for 12 months. Her meals are irregular and consist of dry food. Objectively her diet is sufficient in calories. The tongue is moist with white coating near the root. The abdomen is soft and painful in the epigastrium. What method would be optimal for diagnosis-making in this case?

A. *Esophagogastroduodenoscopy

B. Intragastric pH-metry

C. Fractional gastric analysis (Fractional test meals)

D. Phase-contrast X-ray imaging

E. Biochemical blood test

 

289. A 16-year-old male has been complaining of marked weakness, low appetite, and nausea for the last year, year and a half. Within the last year he has lost 10 kg of body mass. The skin is darkened, especially on his face, neck, and arms. Skin folds and nipples are hyperpigmented, there are pigment spots on the patient’s inner thighs. Pulse is 60/min., blood pressure is 80/50 mm Hg. What is the provisional diagnosis?

A. *Adrenal insufficiency

B. Diabetes mellitus

C. Chronic gastritis

D. Hemochromatosis

E. Cholestatic hepatitis

 

290. A 5-year-old child that contacts with viral hepatitis in the kindergarten presents with increased body temperature up to 38 degrees Celsius, weakness, low appetite, single case of vomiting, dull pain in the subcostal area on the right. The child is provisionally diagnosed with viral hepatitis. What examination would be the most informative for diagnosis confirmation?

A. *ALT activity in blood

B. Urine analysis for bile pigments

C. Feces analysis for stercobilin

D. Blood test for bilirubin

E. Thymol turbidity test

 

291. An infant is 2.5 months old. The onset of the disease was gradual, the child had normal body temperature but presented with slight cough. Within a week the cough intensified, especially at night; on the 12th day the child developed cough fits occurring up to 20 times per day and followed by vomiting. There was one instance of respiratory arrest. Make the diagnosis:

A. *Pertussis

B. Parainfluenza

C. Congenital stridor

D. Respiratory syncytial infection

E. Adenovirus infection

292. A 14-year-old boy presents with moderate bronchial asthma in its exacerbation period. What drug should be prescribed to stop an acute attack of expiratory dyspnea?

A. *Salbutamol

B. Cromolyn sodium (Cromoglicic acid)

C. Dexamethasone

D. Lasolvan (Ambroxol)

E. Strophanthine (cardiac glycosides)

293. A newborn with gestational age of 31 weeks presents with hypotonia and depressed consciousness. Hematocrit is 35%, cerebrospinal fluid analysis shows increased content of erythrocytes and protein, and low glucose. These data correspond with the clinical presentation of:

A. *Intracranial hemorrhage

B. Meningitis

C. Sepsis

D. Anemia

E. Prenatal infection

294. Mother of an 8-year-old girl complains that the child is too short and has excessive body weight. Objectively: obesity with fat deposits on the torso and face (round moon-like face), acne, striae on the thighs and lower abdomen, hirsutism. What hormone can cause such symptoms, when in excess?

A. *Cortisol

B. Thyroxin

C. Testosterone

D. Insulin

E. Glucagon

295. A newborn has Apgar score of 9. When should the infant be put to the breast?

A. *In the delivery room

B. After 12 hours

C. After 2 hours

D. On the 2nd day

E. On the 3rd day

296. A woman with blood group B(III) Rh(+) gave birth to a full-term healthy boy. Examination on the 3rd day of the infant’s life shows him to have icteric tint to his skin. The child has no problems with suckling, sleep is nondisturbed. The abdomen is soft, the liver protrudes by 2 cm from under the costal margin. Complete blood count: hemoglobin – 200 g/L, erythrocytes – 5.5 • 1012/L, total bilirubin – 62 ?mol/L, indirect bilirubin – 52 ?mol/L. What condition can be suspected?

A. *Physiologic jaundice

B. Congenital hepatitis

C. Hemolytic disease of the newborn due to Rh incompatibility

D. Biliary atresia

E. Hemolytic disease of the newborn due to ABO incompatibility

297. A 10-year-old girl started feeling weakness, vertigo, rapid fatigability during the last year. Recently she has developed dyspnea and paresthesia observed. Objectively: skin and mucous membranes are pale and icteric. Lingual papillae are smoothed out. Liver and spleen are at the edge of costal arch. Blood test: Hb- 70 g/L, erythrocytes – 1.7 x 1012/L, blood color index – 1.2, macrocytes. What drug can be prescribed on pathogenetic grounds?

A. *Vitamin B12

B. Vitamin B6

C. Ascorbic acid

D. Iron preparations

E. Vitamin B1

298. A 3-day-old infant with hyperbilirubinemia (428 ?mol/L) developed disturbances manifesting as periodical excitation and convulsions against the background of inertness, hypotension, hypodynamia, and inhibition of unconditioned reflexes, convergent strabismus, rotational nystagmus, and “sunset” eye phenomenon. What is the most likely cause of such symptoms?

A. *Bilirubin encephalopathy

B. Craniocerebral injury

C. Brain tumor

D. Hydrocephalus

E. Infantile cerebral paralysis

 

299. A girl undergoing in-patient treatment for viral hepatitis type B developed headache, nausea, recurrent vomiting, memory lapses, flapping tremor of her hands, rapid pulse. Sweet smell from the mouth is detected. Body temperature is 37.6 degrees Celsius, heart rate is 89/min. What complication developed in the patient?

A. *Acute liver failure

B. Ischemic stroke

C. Gastrointestinal hemorrhage

D. Hypoglycemic shock

E. Meningoencephalitis

 

300. After overexposure to cold a 15-year-old female developed acute pain in her suprapubic and lumbar areas during urination, sharp pains at the end of urination, false urges to urinate. Urine is turbid with blood streaks. The doctor suspects urinary tract infection. What results of laboratory analysis would be the most indicative of such infection?

A. *Leukocyturia, gross hematuria

B. Gross hematuria

C. Increased blood creatinine and blood urea

D. Daily proteinuria under 3.0

E. Daily proteinuria over 3.0

 

301. A 1.5-month-old child on breastfeeding presents from birth with daily vomiting, irregular liquid foamy feces, and flatulence, which are resistant to antibacterial and probiotic therapy; no increase of body mass is observed. The child’s condition improved, when breastmilk was substituted. What pathology is it?

A. *Lactase deficiency

B. Intestinal lambliasis (Giardiasis)

C. Infectious enteritis

D. Drug-induced enteritis

E. Functional dyspepsia

 

302. A 5-year-old child has body temperature risen up to febrile numbers, suffers from inertness, weakness. Examination revealed hemorrhage on the skin of limbs and torso. Enlargement of cervical and axillary lymph nodes can be detected. The liver is 4 cm below the costal arch; the spleen is 6 cm below the costal arch. Blood test: erythrocytes – 2.3×1012/L, Hb – 60 g/L, platelets – 40x 109/L, leukocytes – 32.8 x109/L, eosinophils – 1%, band neutrophils – 1%, segmented neutrophils – 12%, lymphocytes – 46%, monocytes – 1%, blasts – 40%, Duke’s bleeding time test result is 9 min. What examination is necessary to make the diagnosis?

A. *Myelogram (bone marrow biopsy)

B. Lymph nodes biopsy

C. Abdominal US

D. Detection of hepatitis markers

E. Analysis of dynamic platelet function

 

303. An adolescent female complains of weight gain, chills, edema, xeroderma, somnolence, difficulties with focusing. Objectively: height is 165 cm; weight is 90 kg; body proportions are of female type, to- 35,8oC, heart rate – 58/min., BP105/60 mm Hg. Heart sounds are weakened, bradycardia is observed. Other internal organs have no alterations. Thyroid gland cannot be palpated. Milk secretion from mammary glands is observed. Hormone test revealed increased levels of thyroid-stimulating hormone (TSH) and prolactin, and decreased level of thyroxine (4). What is the cause of obesity?

A. *Primary hypothyroidism

B. Secondary hypothyroidism

C. Prolactinoma

D. Hypopituitarism

E. Adiposogenital dystrophy

 

304. An 8-year-old girl with complaints of painful urination, frequent low-volume urination, and leukocyturia was diagnosed with acute cystitis. 10 days before the disease onset she was treated by the gynecologist for acute vulvitis. 5 days ago she presented with mild catarrhal symptoms. Her mother ascribes the child’s disease to her overexposure to cold. Specify the most likely infection route:

A. *Ascending

B. Descending

C. Hematogenic

D. Contact

E. Lymphogenic

 

305. A 5-year-old boy complains of severe dyspnea and sensation of lack of air. Objectively the child assumes orthopneic position, presents with marked peripheral cyanosis, drumstick fingers, nail plates resembling a “clock face”, the borders of cardiac dullness are bilaterally extended; coarse systolic murmur can be detected over the whole surface of the heart and is especially pronounced in the II intercostal area on the left near the sternum. What disease can be characterized by such presentations?

A. *Tetralogy of Fallot

B. Dilated cardiomyopathy

C. Ventricular septal defect

D. Primary bacterial endocarditis

E. Atrial septal defect

 

306. During winter epidemics of influenza caused predominantly by virus А/California/04/2009 (H1N1), on the 2nd day after the disease onset a 10-year-old hospitalized boy presented with high fever, dry cough, myalgia, headache, and general weakness. What should be prescribed as etiotropic treatment in this case?

A. *Neuraminidase inhibitors (Oseltamivir)

B. Antibiotics

C. Immunoglobulin

D. Interferon inducers

E. Acyclovir

 

307. An 8-year-old child presents with blood pressure up to 180/100 mm Hg in the upper limbs accompanied by headaches, tinnitus, occasional nosebleeds, and high fatigability. On examination there is no pulse over the leg arteries. ECG shows left ventricular hypertrophy. MRI-scan shows aortic narrowing to 5 mm in the typical place. Coarctation of aorta is diagnosed. What kind of treatment should be prescribed in this case?

A. *Surgical

B. Conservative

C. Physical therapy

D. Case monitoring

E. Abstain from surgery in favor of complex conservative therapy

 

308. A 15-year-old girl suffers from systemic lupus erythematosus and has been receiving prednisolone therapy in the daily dosage of 2 mg/kg for the last 6 weeks. The plans are made to gradually lower the dosage of the medicine. No clinical signs of her disease are observed. Previously she has received no immunization against measles. Due to measles outbreak it is necessary to develop the immunity against this infection in the patient. When can she be vaccinated?

A. *At the present time

B. After 2 weeks of prednisolone therapy in the dosage lower than 2 mg/kg/24 hours

C. 1 month after the prednisolone therapy is complete

D. Immediately after the prednisolone therapy is complete

E. Never due to medical contraindications

 

309. A 45-year-old woman has been suffering from rheumatoid arthritis for 10 years and takes methotrexate twice a week. What statement regarding vaccination against pneumococci (23-valent vaccine) would conform to the recommendations for the management of rheumatoid arthritis issued by the European League Against Rheumatism in 2010?

A. *Vaccination is recommended

B. Vaccination is not recommended

C. Vaccination is contraindicated to the patients who take methotrexate

D. Vaccination is contraindicated in cases when inflammatory process is active

E. Vaccination necessitates increase in the dosage of the long-term medicines

 

310. A 65-year-old woman was diagnosed with the following: chronic rheumatic heart disease, I degree of rheumatic activity; combined mitral heart disease with prevalence of III degree stenosis; heart failure IIA with retained left ventricular ejection fraction, functional class III (NYHA). What tactics of vaccination against respiratory infections should be chosen to provide secondary prevention of exacerbations and to avoid heart failure decompensation in this patient?

A. *Scheduled yearly vaccination against influenza and pneumococci

B. Vaccination should be combined with antibiotic administration

C. Vaccination is contraindicated due to severe heart failure

D. Any vaccination is contraindicated due to elderly age of the patient

E. Any vaccination is contraindicated due to mitral valve disease

 

311. A 69-year-old woman was diagnosed with the following: ischemic heart disease; stable exertional angina pectoris, FC III; heart failure IIA with retained left ventricular ejection fraction, functional class III (NYHA). What vaccine should be chosen for influenza prevention and to avoid destabilization of the patient’s condition?

A. *Type of influenza vaccine is not important

B. Inactivated influenza vaccine (IIV)

C. Recombinant influenza vaccine (RIV)

D. Vaccination is contraindicated due to severe heart failure

E. Vaccination is contraindicated due to elderly age of the patient

 

312. A 60-year-old man presents with subcompensated viral liver cirrhosis (HCV), Child-Pugh class B. What tactics should be chosen regarding the vaccination against influenza in this case?

A. *Scheduled yearly vaccination

B. In case of influenza outbreak

C. Combined with antiviral drugs

D. Contraindicated due to disease progression stage, as shown by Child-Pugh class

E. Contraindicated due to elderly age of the patient

 

313. A 20-year-old student was brought to the first-aid center. He has a closed fracture of the left forearm and a contused lacerated wound on his left shin. After the patient received initial wound management, he presented the documents confirming that he has received all the necessary preventive vaccination as scheduled. What should the doctor do to prevent tetanus in this patient?

A. *Dynamic case monitoring

B. Administration of tetanus immunoglobulin

C. Administration of anti-tetanus serum

D. Antibiotic therapy

E. Administration of tetanus toxoid

 

314. A 17-year-old girl has made an appointment with the doctor. She plans to begin her sex life. No signs of gynecological pathology were detected. In the family history there was a case of cervical cancer that occurred to the patient’s grandmother. The patient was consulted about the maintenance of her reproductive health. What recommendation will be the most helpful for prevention of invasive cervical cancer?

A. *Vaccination against human papillomavirus (HPV)

B. Vitamins, calcium, omega-3

C. Immunomodulators

D. Antiviral and antibacterial drugs

E. Timely treatment of sexually transmitted diseases

 

315. A 6-month-old infant is not vaccinated. The physician recommends a DPT (diphtheria, pertussis, tetanus) vaccination but the mother is absolutely against this procedure. Choose the most substantial argument in favor of vaccination:

A. *Risk of lethal consequences

B. Epidemic risk for the others

C. Personal professional experience

D. High quality of vaccines

E. –

 

316. In autumn a 45-year-old man was recommended an elective surgery for coronary artery bypass grafting due to multivessel coronary artery disease. The patient has never received anti-influenza vaccination. Why would the family doctor offer a scheduled yearly vaccination against influenza to this patient?

A. *Decrease the risk of mortality due to pneumonia and heart failure

B. Secondary prevention of exacerbations of chronic ischemic heart disease

C. Immunoprophylaxis of postoperative pulmonary complications

D. Primary prevention of influenza during postoperative care

E. Prevention of seasonal influenza

 

317. A 28-year-old woman has made an appointment with the family doctor to receive vaccination against influenza. However, having collected the patient’s medical history, the doctor claimed this procedure to be absolutely contraindicated for this woman. What anamnestic data is the absolute contraindication to vaccination?

A. *Egg white intolerance

B. Pregnancy at 30 weeks

C. Blood hemoglobin – 109 g/L

D. Body temperature – 37.2 degrees Celsius

E. Psoriasis in the remission phase

 

318. During administration of planned DPT vaccination the child suddenly developed acute anxiety, signs of pain response, dyspnea, grunting respirations, marble skin, cold sweat. Objectively the child’s consciousness is disturbed, heart rate is 150/min., blood pressure is 60/40 mm Hg, heart sounds are muffled. The child was diagnosed with anaphylactic shock. What drug should be administered first?

A. * Epinephrine

B. Lasix (Furosemide)

C. Suprastin (Chloropyramine)

D. Euphylline (Aminophylline)

E. Analgin (Metamizole)

 

319. Among first-year schoolchildren there was a case of measles registered. A 7-year-old boy from the same group was not vaccinated against measles due to refusal of his parents. His clinical history has no cases of measles in the past and is not contraindicatory to immunobiological agents. Choose the most rational tactics of measles prevention in this schoolboy:

A. *Measles-Mumps-Rubella vaccine

B. Isolation for 20 days

C. Antiviral agents

D. Antibiotics

E. Immunomodulators

 

320. A 40-year-old man developed fever up to 37.5 degrees Celsius and macular rash 10 days after the first dose of MMR (Measles-Mumps-Rubella) vaccine was administered. The vaccination was considered necessary as there was a measles outbreak in the city and the patient had not received MMR vaccination in his childhood. Is revaccination with MMR vaccine possible?

A. *Possible

B. Forbidden

C. After a course of glucocorticoids treatment

D. Simultaneously with antihistamines

E. Under supervision in the infectious diseases inpatient ward

 

321. A 26-year-old man is undergoing a regular check-up. One year ago he had a case of tonsillar diphtheria complicated with myocarditis. Presently his condition is satisfactory, no signs of cardiovascular failure; ECG shows first degree atrioventricular block. What vaccine was administered to this man according to his age?

A. *Adsorbed diphtheria tetanus vaccine (modified)

B. Acellular DPT vaccine

C. Tetanus anatoxin

D. Oral polio vaccine (OPV)

E. BCG vaccine

 

Additional Questions from other Bases

165 A 6-year-old child has duodenal ulcer. What antibacterial drug should be co-administered together with metronidazole and De-Nol in order to eradicate Helicobacter pylori infection?

A Amoxicillin

B Tetracycline

C Oleandomycin

D Biseptol

E Sulfadimethoxinum

 

Explanation:

DeNol (Brocades) has been introduced as a new treatment for peptic ulcer. It contains tripotassium di-citratobismuthate. The makers claim that the bismuth is held within a citrate complex in a buffer. In the presence of amino acids and proteins, secondary chelates form, which are precipitated in an acid medium.

Non Bismuth quadruple therapy

166 A baby born after fast labour has palsy of hand muscles. Grasp reflex is absent, as well as hand-to-mouth reflex. Hand sensitivity is absent. What is the most likely diagnosis?

A Dejerine-Klumpke palsy → distal brachial plexus paralysis (C7-8-T1)

B Duchenne-Erb’s palsy → proximal brachial plexus paralysis (C5-6)

C Total lesion of the brachial plexus → both proximal and distal (A+B)

D Muscle paresis

E Bernard-Horner syndrome → oculosympathetic paresis

 

Explanation:

Klumpke’s paralysis is a form of paralysis involving the muscles of the forearm and hand, resulting from a brachial plexus injury in which the eighth cervical (C8) and first thoracic (T1) nerves are injured either before or after they have joined to form the lower trunk.

Image result for Dejerine-Klumpke palsy, inferior distal type

 

10. A 10-year-old boy, who was outdoors in windy and cold weather, developed moderate pain and tingling in his fingers and toes. When he had returned home, his parents noticed that the tips of his fingers and toes were white and their sensitivity was lost. The affected areas are warming up, the fingers are tingling and in pain. Skin pallor changed into redness, tingling stopped, slight itching and swelling of the fingers appeared. Determine the frostbite degree in this child:

A. Perniosis

B. Frostbite of the II degree

C. Frostbite of the III degree

D.* Frostbite of the I degree

E. Frostbite of the IV degree

 

Explanation:

Perniosis is an inflammatory disorder that is triggered by prolonged exposure to cold and damp (humid) conditions. It is a form of inflammation of the small blood vessels (vasculitis) and is characterized by painful, itchy, tender, skin lesions on the lower legs, hands, toes, feet, ears and face.A

Frostbite:

First degree

  1. First degree frostbite is superficial, surface skin damage that is usually not permanent.
  2. Early on, the primary symptom is loss of feeling in the skin. In the affected areas, the skin is numb, and possibly swollen, with a reddened border.
  3. In the weeks after injury, the skin’s surface may slough off.

Second degree

  1. In second degree frostbite, the skin develops clear blisters early on, and the skin’s surface hardens.
  2. In the weeks after injury, this hardened, blistered skin dries, blackens, and peels.
  3. At this stage, lasting cold sensitivity and numbness can develop.

Third degree

  1. In third degree frostbite, the layers of tissue below the skin freeze.
  2. Symptoms include blood blisters and “blue-grey discoloration of the skin”.
  3. In the weeks after injury, pain persists and a blackened crust (eschar) develops.
  4. There can be longterm ulceration and damage to growth plates.

Fourth degree

  1. In fourth degree frostbite, structures below the skin are involved like muscles, tendon, and bone.
  2. Early symptoms include a colorless appearance of the skin, a hard texture, and painless rewarming.
  3. Later, the skin becomes black and mummified. The amount of permanent damage can take one month or more to determine. Autoamputation can occur after two months.

 

169. During the first home visit to a full-term boy after his discharge from the maternity hospital a pediatrician revealed a symmetrical swelling of mammae without skin changes over them, swelling of the scrotum. The body temperature was of 36,5oC. The baby was calm, sucked the mother’s breast actively. What condition should you think of?

A Hormonal crisis of the newborn

B Neonatal mastitis

C Sclerema

D Necrotic neonatal phlegmon

E Congenital adrenal dysfunction

 

170-A full-term neonate weighing 4500 g was born asphyxiated with Apgar score of 4-6 points. During the delivery shoulder dystocia occurred. Neurologic assessment revealed non-focal neurologic symptoms, total flaccid paresis of the upper extremities since the arm was atonic and pronated. Grasping, Babkin’s and Moro’s reflexes were absent. What segments of spinal cord had been affected?

A CV – ThI → ( C5 -6-7-8-T1 ) total brachial plexus paralysis , proximal and distal . ERB’s + KLUMPKE

B CI – CII

C CIII – CIV

D ThI – ThV

E ThVI – ThVІІ

 

3.5/5 - (22 votes)

Leave a Comment

Your email address will not be published. Required fields are marked *

You cannot copy content of this page